Download as pdf or txt
Download as pdf or txt
You are on page 1of 52

BIOLOGY

SOLUTIONS 10
Revised Syllabus for Year 2023-2024 (Code No. 086)

Chapter 5 01 - 07
Life Processes

Chapter 6 08 - 16
Control & Coordination

Chapter 7 17 - 27
How do we Reproduce

Chapter 8 28 - 40
Heredity and Evolution

Chapter 13 41 - 49
Our Environment

YOUR FOCUS BECOMES YOUR FUTURE


NAME OF STUDENT :
COACHING BATCH NUMBER:
PHONE NUMBER:
EXAM PREPARING FOR:
© Copyright Reserved.

ALL RIGHT RESERVED BY AUTHOR & PUBLISHER. No part of this book covered by the copyright herein may be
reproduced, transmitted, stored or used in any form or by any means graphic, electronic, or mechanical, including
but not limited to photocopying, recording, scanning, digitizing. taping, web distribution, information networks or
information storage and retrieval system without the prior written permission of the publisher.
BIOLOGY 1

5 ANSWERS & SOLUTIONS


MULTIPLE CHOICE QUESTIONS

1. (d) The small intestine in human beings is the site of complete 16. (d) The exit of unabsorbed food material is regulated by anal
digestion of food. The food gets digested in stomach to sphincter. An anal sphincter is a group of muscles at the
form a semi-solid paste with the help of secretions from liver end of the rectum that surrounds the anus and controls the
and pancreas. Undigested remains then pass onto the large release of stool.
intestine.
17. (c) The contraction and expansion movement of the walls of
2. (c) The small finger-like projections, i.e. villi present in intestine the food pipe is called peristalsis movement.
increase the surface area for better absorption of digested 18. (b) During aerobic respiration, the glucose is converted into
food. pyruvic acid in the cytoplasm of respiring cells.
3. (b) There are three pairs of salivary glands present in humans, 19. (a) Mitochondria performs the cellular respiration in which the
namely the parotid gland, submandibular gland and glucose is broken down to liberate energy in the form of
sublingual glands.
ATP for other metabolic activities.
4. (b) The human saliva contains an enzyme called salivary
20. (a) Muscle cells respire anaerobically to produce lactic acid in
amylase, which breaks down the starch present in food into
the scarcity of oxygen during excessive physical exercise.
sugar. The digestion of starch (carbohydrates) begins in the
mouth. In case saliva is lacking, it will affect the breakdown 21. (b) When air is blown from mouth into test tube, the lime water
of starch. turned milky because the air we breathe out has more CO2.
5. (d) Pancreas secretes pancreatic juice which contains digestive 22. (d) The air for respiration is drawn into our body through
enzymes like amylase, lipases, trypsin. Amylase breaks the nostrils, into nasal passage. From their, air enters into
down the starch, trypsin digests the proteins and lipase
pharynx, larynx, then into the windpipe (or trachea),
breaks down the emulsified fats.
bronchi, lungs and finally to the alveoli where gaseous
6. (c) Proteins present in food are converted to peptones with the exchange takes place.
help of pepsin enzyme. It is secreted by gastric glands found
in stomach wall. 23. (a) Valves ensure that blood does not flow backwards when the
atria or ventricles contract.
7. (b) Parasitic nutrition involves an organism derive its food from
24. (a) The blood leaving the tissues becomes richer in CO2. When
the body of another living organism without killing it.
the oxygenated blood passes through the capillaries of the
8. (d) Fungi obtain nutrients from dead, organic matter, hence body organs/tissues, it gives oxygen to the body cells.
they are called saprophytes. Fungi produce some kind of
At the same time, carbon dioxide produced during
digestive enzymes for breaking down complex food into a

simple form of food. Such, simple form of food is utilized by respiration enters into blood. Thus, deoxygenated blood
fungi. This is defined as the saprophytic mode of nutrition. (carrying CO2) leaves the tissues and enters the vena
cava, from where they are again passed to the lungs for
9. (c) The final product of photosynthesis in plants is glucose and oxygenation.
water. The glucose produced is stored as starch in storage
organs. 25. (d) Arteries carry blood away from the heart to various organs
of the body. Since the blood emerges from the heart under
10. (b) The oxygen gas produced during photosynthesis is released high pressure, the arteries have wall thick ‘and’ elastic walls.
into the surroundings through stomata.
26. (b) Fishes have 2 - chambered heart and blood flows only once
Chlorophyll during a cycle. Group (b) contains fishes, so it represents
11. (c) 6CO2 + 12H2O  → C6H12O6 + 6O2 + 6H2O.
Sunlight correct answer.
12. (d) Plastids structure is involved in the process of formation of
27. (d) Person having blood pressure 140/90, shows hypertension
glucose.
or high blood pressure. The normal blood pressure for
13. (c) Chloroplasts contain chlorophyll which aids in the process humans is 120/80.
of photosynthesis.
28. (c) Arthritis is the inflammation of joints causing pain and
14. (a) When the guard cells become turgid, the stomata open. stiffness.
15. (b) The liver is the largest gland in the body. The liver mainly 29. (a) In rooted plants, transport of water and minerals occurs
secretes ‘Bile Juice’ which is stored in the Gall bladder. through xylem.
2 Chapter 5 : Life Processes

30. (b) Flame cells are the excretory organs of organisms belonging 12. The plant hormones that have been discovered so far are the
to phylum Platyhelminthes. auxins, the gibberellins (GAs), the cytokinins, ethylene, abscisic
31. (a) Kidney is the main excretory organ of human beings, while acid (ABA), jasmonic acid, the brassinosteroids, salicylic acid
other act as accessory excretory organs. (SA), and the bioactive oligopeptides (such as CLE peptides).
13. The process of removal of waste and extra water from blood
32. (c) Nephron is the structural and functional unit of kidney.
is called dialysis [1]. It is an artificial replacement of kidney
33. (a) An adult human produces 1-2 l of urine in 24 hours. This functioning, especially in renal failure cases.
volume depends of intake of fluids physical activities etc.
34. (c) The dialysing fluid has the same composition as that of
blood plasma except that it is devoid of nitrogenous waste
2 MARK QUESTIONS

such as urea. 1. Respiration is a catabolic process in which glucose is broken


down to release energy while photosynthesis is an anabolic
35. (d) All of the given options are examples of liquid waste
products in plants. process in which glucose and other organic substances are
manufactured from raw materials with help of solar radiations.
36. (b) Resins are used in manufacturing of varnishes glazing
2. Ventricles have to pump blood forcefully so as to reach even
agents, etc.
distant capillaries, right ventricle into lungs and left ventricle to

1 MARK QUESTIONS all the remaining body parts, while atria are to pump blood into
adjacent ventricles.
3. (i) During daytime, transpiration pull plays an important role
1. Through its moist skin.


in transportation of water in plants.


2. Aerobic respiration is a two step process. First, glycolysis takes
(ii) At night, root pressure plays an important role in
place in the cytoplasm of the cell where glucose breaks down to
transportation of water in plants.
form pyruvate. Then, pyruvate further breaks down into carbon
dioxide, water and energy, which occurs in the mitochondria. 4. In vertebrates respiratory pigment is haemoglobin. The
respiratory pigments take up oxygen from the lungs and carry it
3. Pisces (Fishes) have a 2-chambered heart. Amphibians have a
to the tissues, which are deficient in oxygen before releasing it.
3-chambered heart. Protozoa do not have a heart. Aves have a
4-chambered heart. Animals have large body size due to which the diffusion pressure
alone cannot deliver oxygen to all parts of the body.
4. Just below the rib cage, one on each side of spine.
5. Osmoregulation is the phenomenon of maintenance of optimum
5. To measure blood pressure, your doctor uses an instrument call a concentration of water and salts in the body fluids. Since, the
sphygmomanometer, which is more often referred to as a blood reabsorption of water from filterate occurs in the kidney tubule,
pressure cuff. The cuff is wrapped around your upper arm and it maintains the water balance and helps in osmoregulation.
inflated to stop the flow of blood in your artery
6.
7.
Translocation of food.
Autotrophs obtain CO2 from air and N2 as nitrate or ammonium
3 MARK QUESTIONS

ion from soil. 1. Trachea is a tube that connects pharynx with lungs for carrying
air to and from lungs. Trachea is lined by ciliated mucus
8. Valves are present in heart and veins to ensure that blood
secreting pseudostratified epithelium for trapping dust particles
does not flow backward, i.e. to allow flow of blood only in one
and microbes. Trachea does not collapse in reduced air pressure
direction. due to support of C-shaped cartilaginous rings.
9. Diffusion from the body surface into surrounding is used by 2. Peristaltic movement or peristalsis is a wave of alternate
single-celled organisms for all vital life processes, e.g. nutrition, contraction and expansion that passes through the gut from
respiration, excretion, etc. oesophagus to large intestine. It moves the food forward in a
10. Transportation of water and minerals regulated manner along the digestive tract for processing in each
11. Ultrafiltration, selective reabsorption, and passive absorption part properly.
all occur in the kidneys during the process of urine formation. 3. The amount of water reabsorbed by nephron depends on two
Ultrafiltration means the diffusion of blood, including plasma to major factors:
the nephron to be filtered. Red blood cells and plasma proteins (i) Amount of excess water present in the body.
cannot pass through the ultrafiltration membrane because their (ii) Amount of dissolved waste to be excreted out of the
size is too big. body.
Reabsorption includes active and passive transport. Water


is absorbed by osmosis or via aquaporin. Ions (e.g.: Na+) are


reabsorbed by active transport while glucose, amino acids, and
small hydrophobic molecules can be reabsorbed by diffusion or
facilitated diffusion. The process of reabsorption occurs in the
renal tubule.
BIOLOGY 3

4. Two differences between arteries and veins is as follows: (iii) The aerobic oxidation of pyruvate occurs in the
S. No Arteries Veins mitochondria; the 3 carbon pyruvate molecule gives rise
1. They carry oxygenated They carry to carbon dioxide and water.
blood (blood that deoxygenated blood Cytoplasm
Glucose  Mitochondria
→ Pyruvate → CO2
In oxygen
carries a large amount (blood that carries


of oxygen) rich in a large amount of  + H2O + Energy


nutrients. carbon dioxide) rich in
3. Yes, it is true that some plant wastes are useful for human beings
waste materials.
for the following reasons:
2. They carry blood They carry blood from
from the heart to the the different parts of (i) Plants like oak store their excretory products as tannin in
different parts of the the body to the heart. trunk which is used in treatment of leather.
body
(ii) Essential oils excreted by plants like sandalwood oil, olive
5. (i) Cramps develop in muscles during excessive activity due oil, eucalyptus oil, etc. are widely used in household
to formation and accumulation of lactic acid as a result purposes.
of lesser oxygen availability than required by muscles.
(iii) Gums from plants are widely used to make adhesives.
(ii) Anaerobic respiration.
(iv) Resins are used to make varnishes, glazing agents, etc.

5 MARK QUESTIONS (v) Natural rubber obtained from rubber plant is used as raw
material in tyre industry.
1. Lymph does not contains oxygen whereas blood does. Lymph 4. (a) Even after forceful exhalation, a large quantity air called
is composed of WBCs and leaukocytes wheras blood contains residual volume is left in the lungs. It is helpful in continuous
haemoglobin, RBCs and WBCs. Lymph is a part of the open exchange of gases.
circulatory system whereas blood is the part of closed circulatory (b) Leakage will result in loss of blood from circulatory system,
system. loss of efficiency of pumping system and non-supply of fresh
2. In the first step of breakdown of a 6 carbon molecule of glucose, blood to many organs. It results in death. To avoid it, blood
a 3 carbon molecule of pyruvate is formed. This process takes platelets release thromboplastin in the region of leakage. It
place in the cytoplasm of the cell. helps in sealing the place of injury through clotting of blood.

Further oxidation of pyruvate depends on the absence or (c) Opening and closing of stomata take place by gain and loss
presence of oxygen as follows: of turgidity in the guard cells.

(i) In yeast, pyruvate is converted into ethanol and carbon 5. (a) Translocation is passage of food materials in solution form
in plants from the region of their supply or manufacture to
dioxide in the absence of oxygen.
the region of their use or storage.
Cytoplasm Absence of oxygen
Glucose  → Pyruvate  → Ethanol


Yeast In plants the region of manufacture of food is only foliage


 + CO2 + Energy while food is required by the whole plant including deep
root tips and top buds and flowers. Similarly regions of
(ii) In our skeletal muscles, Pyruvate is converted into lactic
storage are also away. Therefore, a translocation is always
acid, when there is deficiency/lack of oxygen.
required.
Cytoplasm (In muscle cells) (b) The major areas where translocated substances are delivered
Glucose  → Pyruvate 
Absence of oxygen

are storage regions, growing regions, ripening fruits, etc. Of
 Lactic acid + Energy course, every living cell requires the translocates.

HOTS QUESTIONS

1. Viruses do not show movements outside the host cells. They 4. The pancreas secretes a hormone insulin which regulates
show movement at the molecular level inside the living cells.
the blood sugar level. The hormone secreted by pancereas is
2. Multicellular organism require special organs for exchange of
gases as most of their are internal and not in direct contact with glucagon.
environment.
3. The aquatic organisms obtain oxygen dissolved in water. As 5. The blood circulation in human heart is known as double
compared to air, the availability of oxygen in water is very circulation because the blood passes through the heart twice in
low. Therefore, the aquatic organisms have to breathe faster as
compared to terrestial organisms. one complete cycle of the body.
4 Chapter 5 : Life Processes

CASE BASED QUESTIONS

I. (1) (c) Transportation (12) (d) The most important function of chlorophyll is to


(2) (b) The blood pumping organ is heart. perform photosynthesis in the presence of sunlight
(3) (c) Oxygenated blood from lungs enters left (13) (b) The element is not present in chlorophyll is calcium.


atrium through pulmonary vein.


(14) (c)  Process by which plants prepare their food is called
(4) (b) Deoxygenated blood leaves through the right ventricle


photosynthesis.
through pulmonary artery.
IV. (15) (a) Tissue that transports products of photosynthesis from
II. (5) (a) Yeast


the leaves when phloem.


(6) (b) Ethanol
(16) (a) Sucrose prepared by leaves
(7) (a) Carbon dioxide gas
(17) (c) Xylem in plants is responsible for the transport of
(8) (c) Fermentation


Absorb water and minerals from the soil.


(9) (b) Anaerobic respiration
(18) (b) The two kinds of elements present in xylem are


III. (10) (a) Autotrophic is mode of nutrition opted by plants. tracheids and vessels.
(11) (b) Plants are green in colour because they reflect green


(19) (d) In angiosperms, water is conducted through both


light and they contain a pigment called chlorophyll.


tracheids and vessels.

NCERT QUESTIONS

1. Multicellular organisms such as human beings have complex (ii) Water is taken up or is absorbed by the roots through
body designs and large body size. Thus, they bear specialised the process of osmosis and is transported to the leaves
cells and tissues for performing various necessary functions of the containing photosynthetic cells by the xylem vessels.
body such as intake of food and oxygen etc. Unlike, unicellular 6. Hydrochloric acid (HCl) is the acid secreted inside the stomach
organisms, multicellular organisms do not have all the cells of the and plays the following roles.
body in direct contact with the environment. Hence, diffusion
It is important for the body because:
cannot meet their oxygen requirements as per their body needs.
(i) It makes medium inside the stomach acidic, which is
2. Visible movements like growth of organisms, green colour of
necessary for the activation of enzyme called pepsin. It
plants or running of animals etc.,acts as the main criteria to
converts inactive pepsin into active pepsin.
decide whether something is alive or not.
(ii) It also kills any bacteria, entering the stomach along with the
3. Carbon based molecules, i.e. food is used by body from outside
food.
to meet its energy need. Oxygen is used to oxidise food which
release energy. So, food and oxygen are the basic raw materials 7. Digestive enzymes help to breakdown large, insoluble food
used by an organism. molecules into small water soluble, molecules, e.g. enzyme
amylase breaks down starch and enzyme trypsin helps to
4. Four basic processes that are essential for maintaining life are: breakdown proteins.
(i) Nutrition It is the process of transferring source of energy 8. (d) All of these
from outside to the body of an organism.
9. The small intestine is the site of the complete digestion of fats.
(ii) Respiration It is the process of acquiring oxygen from The upper part of the small intestine receives bile Juice, which
outside into the body and using it for the breakdown of contains bile salts for breakdown of fats into smaller globules
food sources to release energy for cellular needs. thereby, increasing the efficiency of the enzyme action. This
(iii) Transportation It is the process of carrying food and oxygen process is known as emulsification.
from one place to another in the body. 10. Saliva is secreted by the salivary glands in the mouth. It contains
(iv) Excretion It is the process of removing byproduct and the enzyme salivary amylase, which breaks down starch into
waste products from body which are formed during energy maltose.
generating reactions. 11. The necessary conditions for autotrophic nutrition are:
5. The raw materials for photosynthesis are carbon dioxide and (i) water (ii) carbon dioxide
water. These are taken up by the plants in the following (iii) sunlight (iv) chlorophyll
ways:

The byproduct of autotrophic nutrition is oxygen, which is


(i) CO2 is taken up through the stomata from the atmosphere


released into the atmosphere through stomata.


in case of land plants, while the aquatic plants take up CO2
dissolved in water.
BIOLOGY 5

CBSE PAST YEAR QUESTIONS

1. The differences between autotrophic and heterotrophic nutrition 5. Amoeba is a unicellular, omnivore organism. The mode of
nutrition in Amoeba is holozoic.It takes place with the help of
are as follows:
pesudopodia (finger-like extension).It engulfs the food when it
comes in contract with its cell surface to form a food vacuole.
Features Autotrophic Heterotrophic
Nutrition Nutrition
Nucleus
Food In this mode of In this mode of
Food particle
nutrition, food nutrition food is (a) Ingestion
is prepared by obtained from other Pseudopodia
organism itself. organisms
(autotrophs).
Food vacuole
(b) Digestion Food particle
Inorganic The raw materials The raw materials are
substances are required by not required.
the organism Undigested food
(autotrophs).
Removed
(c) Absorption
Digestion This process is This process is
absent. required for the
conversion of complex
molecules (d) Egestion
into simpler and more
soluble ones.
Figure: Different stages of nutrition in Amoeba

Chlorophyll It is present in It is absent. 6. (i) The cells of liver secrete bile juice which helps in
autotrophs for emulsification of fats. It also makes the acidic food coming
trapping light. from the stomach alkaline thus, facilitating the action of
pancreatic enzymes.
Status They are known as They are known as
producers. consumers. (ii) Gall bladder stores and concentrates bile juice produced by
the liver.

2. The small intestine is the main region for the absorption of The villi increases the intestinal absorptive surface area. This
ensures that there is more space for the food and its components
digested food. The inner lining of small intestine is covered by to be absorbed.
millions of tiny finger-like projections called villi. The presence 7. (i) Nutrition is necessary because
(a) it helps in the growth of new cells and repair of older
of villi gives the inner walls of the small intestine a very large ones.
(b) it is needed to develop resistance against diseases.
surface area for the absorption of digested food. (c) it provides energy for metabolic processes.

3. Gastric glands are found in the wall of the stomach that release (ii) Disappearance of green plants from Earth would mean a
total disaster for the ecosystem. Green plants are the source
secretions for the digestion of food. of energy for all organisms. All other organisms directly or
indirectly depend on them for food. So, if they disappear
Three components of secretion released by these glands are: from the Earth, all the herbivores will die due to starvation
and so will the carnivores. It would result in the extinction of
(i) Mucus
life from the Earth.
8. (i) HCl kills bacteria in the stomach and provides acidic
(ii) Hydrochloric acid medium for the activation of pepsinogen into pepsin.
(enzyme required for protein digestion).
(iii) Pepsinogen (ii) (a) The release of food from the stomach to small intestine


is controlled by gastric sphincter.


4. Glucose and amino acids are the final products obtained after
(b) The release of undigested food from rectum through


digestion of carbohydrates and proteins, respectively. the anus is controlled by anal sphincter.
6 Chapter 5 : Life Processes

9. Carbohydrate digestion begins in the mouth or buccal cavity. 13. Haemoglobin is the red pigment which carries oxygen in the
The salivary glands in the mouth secrete saliva, which helps to blood. RBC contain haemoglobin in it. Haemoglobin is an
moisten the food. The food is then chewed while the salivary iron-containing protein that carries oxygen from the lungs
glands also release the enzyme salivary amylase, which begins to the body14. Nephrons are the basic filtering units
the process of breaking down the carbohydrate food. of kidneys. Each kidney possesses large number of nephrons,
10. Valves are flap-like structures which prevent backflow of blood approximately 1-1.5 million. The main components of the
inside the heart during contraction nephron are glomerulus, Bowman’s capsule, and a long renal
tubule.
11. Amoeba has contractile vacuoles that help in excretion.
Earthworm have nephridia for excretion. 15. Dialysis is the process of removing excess water, solutes, and
Cockroach have malpighian tubal system for excretion. toxins from the blood in people whose kidneys can no longer
perform these functions naturally.
12. The artery carries oxygenated blood away from the heart.
The pulmonary artery is the only artery that carries the

16. Transpiration is the process when plants loose water as vapour.
deoxygenated blood to the lungs for oxygenation. 17. During excretion in human beings, glucose which enters the
The opening of the pulmonary artery is guarded by the
 nephron along with filtrate gets reabsorbed by blood capillaries
pulmonary valve, formed by three semilunar cusps. surrounding the nephron.

NTSE & OLYMPIAD QUESTIONS

1. (a) The process of repairing and maintaining requires molecular 11. (c) Gastric juice is a strong acidic liquid, pH 1 to 3, which is
movements in life. When there is a bleeding from the cut close to being colourless. The hormone gastrin is released
finger, these molecular movements not only helps to stop into the bloodstream when peptides are detected in the
the bleeding but also helps in healing the wounds. stomach. This causes gastric glands in the lining of the
2. (b) Enzymes are proteins that catalyze chemical reactions. stomach to secrete gastric juice. Its main components are
Almost all processes in a biological cell need enzymes in digestive enzymes pepsin and rennin, hydrochloric acid,
order to occur at significant rates. The set of enzymes made and mucus.
in a cell determines which metabolic pathways occur in that 12. (c) In cell biology, a mitochondrion is a membrane-enclosed
cell. organelle, found in most eukaryotic cells. Mitochondria are
3. (d) Earthworm respires, but has no respiratory organs, “cellular power plants,” because they generate most of the
exchange of gases takes place through moist skin. cell’s supply of ATP, used as a source of chemical energy.
Each pyruvate molecule produced by glycolysis is actively
4. (d) In lungs, exchange of gases takes place in Alveoli
transported across the inner mitochondrial membrane,
5. (b) The gaseous exchange is the simple diffusion of oxygen and into the matrix where it is oxidized and combined with
from alveolar air into the blood and diffusion of CO2 from coenzyme A to form CO2, acetyl-CoA and NADH.
blood to alveolar air.
13. (a) Nature of valves in the heart is membranous
6. (c) Photosynthesis uses light energy and carbon dioxide to 14. (a) Heart beat can be initiated by Sinu-auricular node.
make triose phosphates (G3P). G3P is generally considered
15. (a) The left atrium receives oxygenated blood from the lungs
as the prime end-product of photosynthesis. It can be used
through two parts of pulmonary veins.
as an immediate food nutrient, or combined and rearranged
to form monosaccharide sugars, such as glucose, which can 16. (a) Small intestine is the largest part of the human alimentary
be transported to other cells, or packaged for storage as canal whose length is approximately 7.2 m.
insoluble polysaccharides such as starch. 17. (b) De-oxygenated blood enters the right atrium of the heart
and flows into the right ventricle where it is pumped through
7. (a) Saliva is the watery and usually frothy substance produced
the pulmonary arteries to the lungs. Pulmonary veins return
in the mouths of humans and some animals. In animals,
the now oxygen-rich blood to the heart, where it enters the
saliva is produced in and secreted from the salivary glands.
left atrium before flowing into the left ventricle. From the left
Saliva contains the enzyme amylase that breaks down some
starches into maltose and dextrin. Thus, digestion of food ventricle the oxygen-rich blood is pumped out via the aorta,
occurs within the mouth, even before food reaches the and on to the rest of the body.
stomach. 18. (a) Largest heart is found in Elephant.
19. (a) Cardiac impulse (Heart beat) normally originate from the
8. (d) In anaerobic respiration the biproduct is carbondioxide.
pacemaker (S.A. node).
9. (c) The blood returning to the heart from lungs via pulmonary
20. (c) Blood oxygenation is measured in several ways, but the
vein has more oxygen per ml of blood
most important measure is the haemoglobin (Hb) saturation
10. (c) The first heart sound is ‘Lubb’ sound at the beginning of percentage. The haemoglobin molecule is the primary
systole transporter of oxygen in mammals.
BIOLOGY 7

21. (b) A sphygmomanometer or blood pressure meter is a 39. (c) A – (s); B – (t); C – (p); D – (r); E – (q)
device used to measure blood pressure, comprising an 40. (a) A – (s); B – (r); C – (q); D – ( p)
inflatable cuff to restrict blood flow, and a mercury or 41. (a) A – (q); B – (p); C – (s); D– (r)
mechanical manometer to measure the pressure. Manual
42. (c) A – (s); B – (r); C – (p); D – (q)
sphygmomanometers are used in conjunction with a
43. (a) A – (p); B – (s); C – (q); D – (r)
stethoscope.
44. (b) We can detect the malfunctioning of heart by
22. (a) Transpiration is the evaporation of water from aerial parts
Electrocardiogram. It records the electrical activity of heart.
and of plants, especially leaves but also stems, flowers and
fruits. Transpiration is a side effect of the plant need to open 45. (c) Both (i) and (ii) are correct
its stomata in order to obtain carbon dioxide gas from the 46. (b) Herbivore is an animal that feeds on plants only.
air for photosynthesis. Transpiration also cools plants and 47. (d) Both statements are wrong
enables mass flow of mineral nutrients from roots to shoots. 48. (d) Autotrophs are organisms that are able to synthesize their
23. (d) The main effect of cytokinin in plant is to stimulate cell own food using an external energy source, e.g., plants,
division. green algae and cyanobacteria make their food with the
24. (d) Growth of pollen tube towards ovule is called Chemotropism help of chlorophyll.
25. (a) In medicine, dialysis is a type of renal replacement therapy 49. (b) Plants possess pigment molecules for absorption of light
which is used to provide an artificial replacement for lost energy. These pigments occur in green coloured cell
kidney function due to renal failure. Dialysis may be used oganelles called
for very sick patients who have suddenly lost their kidney 50. (a) Skin is the main excretory organ.
function (acute renal failure) or for quite stable patients 51. (b) Contractile vacuole of Amoeba takes part is digestion of
who have permanently lost their kidney function (end stage food.
renal failure).
52. (a) Urea is a waste products of animals.
26. (d) Plants bend towards a light source as a result of unequal
53. (a) Both A and R are individually true and R is the correct
auxin distribution in the stem.
explanation of A
27. (a) Auxin does not promote the growth of root.
54. (b) The blood of an insect functions differently than the blood
28. (b) Resin is a hydrocarbon secretion of many plants, particularly of a human. Insect blood, however, does not carry gases
coniferous trees, valued for its chemical constituents and
and has no haemoglobin which gives red colour to the
uses such as varnishes, adhesives, as an important source
blood.
of raw materials for organic synthesis, or for incense and
55. (b) Blood pressure, sometimes called arterial blood pressure, is
perfume. Gum is also a byproduct produced and is sent out
the pressure exerted by circulating blood upon the walls of
through the bark of the tree.
blood vessels.
29. (d) Mouth → oesophagus → stomach → small intestine →
56. (c) Bowman’s capsule found in heart which accomodates
large intestine.
one glomerulus, is lined by flat cells some of which have
Cytoplasm Mitochondria
30. (d) Glucose  → Pyruvate →
In oxygen fine pores to allow passage of materials filtered out of a
Ethanol+Carbon dioxide glomerulus.
Cytoplasm
31. (d) Glucose 
→ Pyruvate+Energy 57. (b) Both A and R are individually true but R is not the correct
mitochondra explanation of A.
 → CO 2 + H 2O + Energy
58. (b) Blood vessels which are thin walled and extremely narrow
32. (d) Both (ii) and (iv) statement are true for respiration
tubes or which connect arteries to veins
33. (c) Both (ii) and (iv) statement are true about heart
59. (d) Platelets are the tiny fragments of special cells formed in
34. (b) Plants absorb nitrogen in form of ntrates, nitrites and urea bone marrow and helps in coagulation of blood.
35. (c) (i) and (iv) statement are correct 60. (c) Excreton is the process of removal of toxic wastes from the
36. (c) Kidney → Ureters → Urinary Bladder → Urethra. body of an organism.
37. (a) Water rises in delivery tube only in setup (i) 61. (c) Photosynthesis is the process by which green parts of plant
38. (a) (i) and (iii) will become black when dipped in iodine synthesise organic food in the form of carbohydrates from
solution. CO2 and water in the presence of sunlight.

❑❑❑
8 Chapter 6 : Control and Coordination

6 ANSWERS & SOLUTIONS

MULTIPLE CHOICE QUESTIONS

1. (a) The correct answer is Sensory organ → Spinal cord → 17. (b) The flame represents stimulus. The phenomenon of in-
Effector organ. stantaneous decisions taken by our body without active
2. (c) Synapse is a gap between two neurons. It is a place, thinking is called reflex action.
where information is transmitted from one neuron to the
18. (a) Pancreas acts as both endocrine and exocrine gland.
other neuron.
19. (d) As Iron is not mandatory so above three statement are
3. (d) Brain is the controlling centre of our body. It has differ-
correct.
ent regions responsible for different functions of the body
such as thinking, balancing, regulating, etc. 20. (a) Growth hormone is secreted by Pituitary gland.
4. (b) In neuron, “conversion of electrical signal” to a “chemi- 21. (c) Testosterone from testes and estrogen from ovary.
cal signal occurs at axonal end”
22. (d) Deficiency of Insulin causes diabetes mellitus.
5. (a) Forebrain or cerebrum is the most well-developed part of
the human brain. 23. (c) The Pituitary gland does not exist in pairs.

6. (c) Hypothalamus controls and regulates temperature of 24. (c) Pituitary gland is the ‘master gland’ in our body.
body, urge of eating, drinking, sleeping, etc.
7.
8.
(b)
(b)
Cerebellum controls the voluntary actions, e.g. posture.
Spinal cord begins in continuation with medulla and ex-
1 MARK QUESTIONS

tends downwards. 1. Some sudden action in response to something in the environ-


ment.
9. (b) Motor neurons carry nerve impulses from the brain and
spinal cord to muscles and glands. Interneurons carry 2. The gap between two neurons is called a Synapse.
nerve impulses back and forth between sensory and mo- 3. Iodine stimulates the thyroid gland to produce thyroxin hor-
tor neurons. mone. Deficiency of this hormone results in the enlargement
10. (d) A microscopic gap between a pair of adjacent neurons of the thyroid gland. This can lead to goitre.
over which nerve impulses pass when going from one 4. Auxins are the plant hormones that promote growth. They


neuron to the next is called Synapse. promote stem elongation, stimulate branching in roots and
inhibit the growth of lateral buds in plants. Abscisic acid and
11. (a) Spinal cord.
ethylene inhibit plant growth.
12. (c) The highest coordinating centre in the human body is 5. The most important hormone that the pancreas produces is


brain. insulin. Insulin is released by the ‘beta cells’ in the islets of


13. (a) The cerebrum (part of forebrain) is the main thinking Langerhans in response to food. Its role is to lower glucose
part of the brain. It coordinates the voluntary actions of levels in the bloodstream and promote the storage of glucose
the body. It has different areas for performing different in fat, muscle, liver and other body tissues.
functions such as centres of hearing, smell, memory, etc. 6. Auxins, Gibberellins, Cytokinins, Ethylene and Abscisic acid.
14. (d) Medulla: The primary role of the medulla is regulating 7. Two main parts of hind-brain are — Medulla and Cerebellum.
our involuntary life sustaining functions such as breath- 8. An example of a plant growth promoter is gibberellins and
ing, swallowing and heart rate. As part of the brain stem, example of a plant growth inhibitor is abscisic acid.
it also helps transfer neural messages to and from the
9. Three endocrine glands in human body are Thyroid gland,
brain and spinal cord.
Endocrine gland and Adrenal gland.
15. (b) Spinal cord controls the reflex activities of the body. 10. Chemotropism is the movement of a part of the plant in re-
16. (d) The correct answer is Receptor → Spinal cord → sponse to a chemical stimulus.
Muscle. In the reflex action, the reflex arch is formed by 11. ‘Hydrotropism’ is the directional growth of a plant part in re-
Receptor → Spinal cord → Muscle. A reflex arc is a neu- sponse to water.
ral pathway that controls a reflex action. In vertebrates,
12. Spinal cord is enclosed in a bony cage called vertebral col-
instead of passing directly to the brain, the sensory neu-
umn.
ron’s synapse in the spinal cord.
BIOLOGY 9

13. Testes secrete male sex hormone called testosterone. 2. Pancreas is an important endocrine gland in human body,
14. No reflex action is not only function of spinal cord; it is function which helps in regulating the glucose level in blood by releas-
of central nervous system which consist of spinal cord and brain. ing an important hormone called insulin. It also secretes di-
gestive enzymes such as amylase, lipase etc., which help to
15. In fore-brain hypothalamus is responsible for this feeling.
breakdown various components of food. If pancreas will stop
16. The cerebellum is the area at the back and bottom of the brain, functioning, the person would need to administer insulin in-
behind the brainstem relating to movement and coordination. jections as well as required to take multiple digestive enzymes
17. The plant hormones which help or promote: pills in order to help the breakdown of food.
(i) Cell division — Cytokinins 3. We blink our eyes as bright light is focused on us due to reflex
action. It is done to protect the eye from bright light which
(ii) Growth of the stem — Gibberellins
otherwise would damage the retina. The amount of light that
18. The two main constituents of the Central Nervous System in enters your eyes is controlled by the pupils. Hence, blinking
human beings are the brain and the spinal cord. the eyes cuts off the excessive light.

2
4. Pons and Medulla oblongata are located in hindbrain.
MARKS QUESTIONS
Function of Pons – it acts as a bridge between the brain and
spinal cord.
1. When light falls on the side of the shoot auxin diffuses towards
Function of Medulla oblongata – It controls activities like sal-
the shady side of the shoot. This concentration of the aux-
ivation, swallowing, vomiting, breathing, coughing, sneezing,
in stimulates the cell to grow longer on the side of the shoot
heartbeat, sleep, consciousness and activities of the cerebrum
which is away from light. Thus plant appears to bend towards
light. 5. Nervous and hormonal systems together perform the function
of control and coordination in human beings. Under stressed
2. Some patients of diabetes are treated by giving injections of
conditions, the stimulus is being perceived by the Central Ner-
insulin because insulin hormone regulates the levels of sug-
vous System which stimulates the adrenal gland that secretes
ar in the blood. In diabetic patients, insulin hormone is not
the adrenaline hormone which prepares the body to face the
secreted by pancreas in the required amount and therefore
emergency by increasing the breathing rate and heartbeat so
blood sugar level rises.
that more oxygen can be supplied to the body. The blood sup-
3. Touch–me–not plant responds to touching by folding its leaf- ply decreases from the skin and digestive system and increases
lets and this type of movement is called growth independent to skeletal muscles.
movement i.e., the movement of plants that do not result in

5
their growth.
MARKS QUESTIONS
4. (i) Medulla oblongata in hind brain controls swallowing.

(ii) Cerebrum in forebrain controls hearing. 1. Difference between involuntary and reflex actions is given
5. Phytohormones are synthesized at sites away from where they below:
act. They diffuse to the area of action and help to coordinate S. Involuntary action Reflex Action
growth, development and responses to the environment. Phy- No.
tohormones are –
1. It is the set of muscle It is rapid and spontaneous
Auxin, Gibberellins, Cytokinin, Abscisic acid, Ethylene. movement which do not action in response to any
require thinking. stimulus.

3 MARKS QUESTIONS
2. It is controlled by the It is controlled by the spinal
1. (i) Thyroid gland It secretes thyroxine hormone, which brain. cord.
regulates the metabolism of carbohydrates, fats and pro- 3. Example – breathing, Example- Sudden jerky
teins in body. Iodine is essential for the synthesis and digestion withdrawal of hand after
secretion of throxine. touching something hot.

(ii) Pituitary gland It is known as the master gland of the 2. (i) Thyroxine controls the overall metabolic rate of the body.
human body as it controls the functioning and secre-
tions of other glands. It secretes many hormones, one (ii) Insulin converts glucose to glycogen in liver and mus-
of which is Growth hormone that regulates growth and cles and controls the blood sugar level.
development of bones and muscles. (iii) Adrenaline increases heartbeat and blood pressure.
(iii) Pancreas (Both exocrine and endocrine, ie. mixed (iv) Growth hormones are responsible for body growth
gland) contains specialised cells which secrete two major and development of bones.
hormones (v) Testosterone is the male hormone and develops male
(a) Insulin, which lowers the blood glucose level, sex organs and secondary sex characteristics like mous-
(b) Glucagon, which increases the blood glucose level. tache, beard and voice.
10 Chapter 6 : Control and Coordination

3. Plants also perform chemical coordination for various activi- 5. (a) Excitation of neuron is transferred to muscle at the neu-
ties with the help of hormones. These are the chemical com- romuscular junction by the release of acetylcholine from
pounds released by stimulated cells that diffuse to various the knobs of its motor end plate. Acetylcholine is picked
locations in plants performing different functions. These hor-
up by Ach receptors present on the surface of muscle
mones produced by plants are also called as phytohormones.
Different types of hormones produced by plants are: Auxin, fibre. This induces the mucles fibre to contract.
Gibberellins, Cytokinins, Abscisic acid, Ethylene. (b) Testes in males produces hormone testosterone. Ovaries
4. (a) Tropism: It is the phenomenon of directional paratonic in females produces hormone oestrogen.
growth movement of curvature in which the direction
of movement is determined by the direction of stimulus, (c) A growth movement of a plant in response to an external
e.g., phototropism, geotropism, hydrotropism. stimulus in which the direction of stimulus determines
the direction of response is called tropism.
(b) Protection of Brain,
• Bony box or skull on the outside, (d) Electrical.
• Meninges over the brain.
• Presence of cerebrospinal fluid both outside and
inside the brain to absorb shocks.
(c) Cerebellum

HOTS QUESTIONS

1. There are two hormones that directly affect breastfeeding: 6. In stress situations adrenaline is released into the blood by
(a) Prolactin (Maternity Hormone): Production of milk. adrenal glands. Its target organ is heart which as results heart
rates increases so as to increase the supply of the oxygen to
(b) Oxytocin: Ejection of milk.
the muscles, blood to digestive system and skin is reduced and
2. Dormin is the other name of plant hormone abscisic acid. The breathing rate increases. All these responses together prepare
hormone induces dormancy in buds and seeds. So it has been the body to face any stressful situation.
called dormin.
7. If intake of iodine in our diet is low, it results in a goiter and it
3. Pancreas produces two hormones.
deteriorates the physical condition of the body.
(a) Insulin from b-cells of islet of Langerhans and
8. The hormone feedback mechanism is defined as the mech-
(b) Glucagon from a-cells of islets of Langerhans.
anism that is used to maintain the hormone balance in the
Insulin is produced when glucose level of blood rises. Insulin blood / body. The increase or decrease in the concentration of
helps the cells to withdraw glucose from blood. It also converts
that hormone can sometimes stimulate and increase the secre-
glucose into glycogen in liver and muscles.
tion of that hormone or inhibit the secretion of the hormone.
4. There is a need for the chemical communication in multi-
cellular organisms. These communication is done by differ- 9. The medulla oblongata is a part of the hindbrain and is re-
ent hormones. These hormones are responsible for different sponsible for regulating automatic functions of the body such
functions in the body like digestion, respiration, reproduction, as breathing, heart rate.
metabolism, etc.
10. Muscle cells have special proteins that change both their
5. When light is coming from one direction of plant, auxin syn- shape and their arrangement in the cell in response to nervous
thesised at the shoot diffuses towards shady side. This concen-
electrical impulses. When this happens, new arrangements of
tration of auxin stimulates the cells to grow longer on the side
of the shoot which is away from light. Thus plant appears to these proteins give the muscle cells a shorter form. Thus, the
bend towards light. muscle contracts.

CASE BASED QUESTIONS

I. (1) (a) True III. (10) (a) Phototropism


(2) (a) Axon
(3) (a) Dendrites (11) (b) Geotropism
(4) (b) Synapse
II. (5) (a) Pancreas (12) (c) Chemotropism
(6) (a) Insulin
(7) (a) Diabetes (13) (d) Hydrotropism
(8) (b) Diabetic person
(9) (c) Liver (14) (a) Thigmotropism
BIOLOGY 11

IV. (15) (a) Thigmonasty (20) Forebrain, midbrain and hindbrain are the three areas of
(16) (a) True
(17) (a) True the brain, constituting different parts of the brain. Forebrain
(18) (a) True
(19) (b) Large comprises cerebrum, thalamus and hypothalamus. Midbrain
V.
is located between the thalamus of the forebrain and pons
Forebrain Parietal lobe
of the hindbrain. Hindbrain comprises pons, cerebellum and

Cerebrum medulla.
Occipital lobe

(21) Temporal lobe

(22) Cerebellum

Temporal lobe Midbrain (23) Cerebrum

Cerebellum
Pons (24) Pituitary gland is to produce and release several hormones
Pituitary
Gland
that help carry out important bodily functions.

NCERT QUESTIONS

1. The differences between a reflex action and walking are as 5. Plant hormones are organic substances produced naturally
follows:
in higher plants. They control growth and other physiological
S. No. Reflex Action Walking
functions of the plants. These are required in very small
1. It is an spontaneous and It is acquired through
involuntary response to learning and is a amounts.
a stimulus. voluntary response.
2. It is regulated by spinal It is coordinated by the 6. Differences between movement of leaves and movement of
cord. brain (hindbrain).
shoot towards light are as follows:
3. Its intensity cannot be Its intensity can be
changed. changed.
4. It increases the survival It is concerned with the S. No. Movement of Leaves Movement of Shoot
and protective values of locomotion. (Sensitive Plant)
an organism. 1. This movement is This movement is
independent of growth dependent on growth.
2. Synapse is the small gap between two neurons. At the end of
axon, the electrical impulse sets the release of some chemicals 2. Stimulus is touch. Stimulus is light.
a form of neurotransmitters (acetylcholine) which cross the 3. Movement is not Movement is directional.
gap (synapse) and start a similar electrical impulse in the directional.
dendrite of the next neuron. Synapse actually acts like a one 4. Movement is neither Movement is towards
way valve because the chemical substance is present only at away nor towards the the stimulus.
one side of the gap. stimulus.

3. The smell reaches the neurons of olfactory receptor of nose. It


7. Auxin is a plant hormone, which promotes growth in plants.
causes the generation of nerve impulses that reach the olfactory
lobes of the forebrain to produce the sensation of smell.
8. Auxins promote cell elongation and are present at the shoot
4. Brain has no role to play in reflex action. It is an extremely
quick action, which does not involve any thinking by the tip. When tendril comes in contact with a support, auxin
brain. In this type of action, the stimulus is received by the
stimulates faster growth of the cells on the opposite sides.
spinal cord that sends a response, e.g. coughing. The action is
registered in cerebral brain just for memory. Thus, the tendril coils around the support.
12 Chapter 6 : Control and Coordination

9. The growth of plant parts towards or away from water is called These chemicals cross the synapse and reach the next neuron.
hydrotropism. It is shown in the diagram given below: This is how nerve impulses travel through the body. Thus,
neurons are important in receiving information from the
Water
surroundings and in sending it to the effector.
Porous Pea seedling
pot 16. The directional movement of the plant in response to light is

Sand Hydrotropic called phototropism. The shoots respond by bending toward


curvature light and roots respond by bending away from the light.
To perform the experiment, take a plant (pea seedling) in a This happens as follows:
jar filled with sand. Place a porous pot filled with water in the
(i) When sunlight falls straight on the plant, the auxin hor-
wide jar. Roots of the plant will grow towards water and bend
mone synthesised at the tip of the stem spreads uniform-
towards the water source showing positive hydrotropism.
ly down the stem and due to equal concentration of aux-
10. Chemical coordination is performed by hormones, which are
in, stem grows straight.
chemical messengers secreted by endocrine glands.
(ii) When sunlight falls on only one side of the plant, the
These hormones coordinate the activities of living organisms
and also their growth. auxin diffuses towards the shady side of shoot. The con-
centration of auxin stimulates the cells to grow longer.
The timing and the amount of hormones released are
regulated by the feedback mechanism. Therefore, the stem appears to bend towards the source
of light.
11. Iodine is required for the proper functioning of Thyroid gland.
If its amount is less, than neck will swell up due to iodine
deficiency causing disease Goitre, therefore iodised salt is
advised in diet
12. Adrenaline hormone is secreted from adrenal gland into the
blood and carried to different parts of the body. At the time of
emergency, danger or stress, it is released in large quantities.
As a result, it causes fast beating of heart, resulting in supply
of more oxygen to muscles.
The blood supply to the digestive system and skin is reduced
Phototropism
due to contraction of muscles around small arteries in these
organs. The breathing rate also increases because of the 17. In case of a spinal cord injury, signals for reflex actions and
contraction of diaphragm and rib muscles. All these responses involuntary actions will get disrupted.
together help the animal to deal with the emergency situation.
18. Chemical coordination in plants is achieved by the plant
13. (d) Cytokinin is a plant hormone that promotes cell division
hormones. Plant hormones are the chemical compounds,
in plants.
which help the plant to coordinate the growth, development
14. The function of receptors in our body is very important as
and responses to the environment. Plant contains the following
they collect informations about changes in the environment
around us. Receptors then pass the same information in the hormones:
form of nerve impulse to the central nervous system, where (i) Auxins helps in cell elongation.
the information is processed and the ultimate response is
(ii) Gibberellins helps in the growth of the stem.
given.
15. (iii) Cytokinins promotes cell division.
Axon (iv) Abscisic acid inhibits plant growth.
Nucleus Soma terminal
(cell body) 19. All the organisms need a well-organised system of control
Dendrite
Myelin
and coordination. Responding to stimuli is a characteristic
sheath property of all living organisms. On receiving a stimulus, the
body responds in a manner that is most appropriate for its
survival and functioning. The response that is given involves
various organs (or parts) of body.
Axon
Thus, it is very necessary that all the organs work together in a
Functions of a neuron The neuron receives information
from receptors as electrical impulse, at its dendritic end. The proper coordinated way so as to provide the correct response.
impulse then travels from dendrite to the cell body and further This working together of various organs in a systematic orderly
along axon to its end. At the end of axon, electrical impulse way to provide proper control and response to stimulus is
leads to the release of some chemicals. called coordination.
BIOLOGY 13

20. Comparison between hormonal mechanisms and nervous 7. It is involuntary. It can be voluntary or
mechanism are as follows: involuntary.
It is through chemicals called hormones.
8. Nervous Mechanism
S. No. Hormonal Nervous 9. It is through electrical
mechanism mechanism conduction.
1. The system is The system is rapid. 21. The differences between the movement in sensitive plant and
comparatively slower.
our legs are as follows:
2. The system is not The system is directly
connected to target connected to every part S. No. Movement in Movement in Legs
sites directly. under its control. Sensitive Plant
3. The response is slow The response is quick 1. It occurs in response It is a voluntary action
and produced by and limited to those cells
all the cells of target that are innervated with to an external performed and controlled
tissues. nerves. stimulus like touch. by our will.
4. The system controls It has little role in growth 2. Plant cells change Movement of legs
growth and and development. their shape by (voluntary action) is
development. changing the amount controlled by cerebellum,
5. It consists of glands It consists of neurons, of water (turgor a part of hindbrain. It is
and their secretions. nerves and nervous changes) in them. due to change in special
organs.
proteins of muscles.
6. The effect of chemical The effect of nervous
message lasts for message is for short 3. No nerves are Nerves carry the message
longer period. duration. involved. for movement of legs.

CBSE PAST YEAR QUESTIONS

1. The part of brain that maintains posture and equilibrium is 7. (i) The pathway taken by nerve impulses in a reflex action
Cerebellum. is called the reflex arc. They allow rapid response to a
stimulus, e.g. Pulling of hand on touching a hot object.
2. Deficiency of Iodine causes our neck to swollen due to
the enlargement of thyroid gland causing disease Goitre. (ii) Muscle cells have special proteins that change their shape
and arrangement in the cell in response to electrical
Therefore it is advisable to use Iodine in diet.
impulse. This forces the muscle cells to contract and
3. Two components of central nervous system are relax, causing their movement.
(i) Brain 8. Movements in Mimosa pudica (touch me not) occur in
(ii) Spinal cord. response to touch.
4. The shoots of the plant bend towards the light, whereas roots In such movements, plant cells change shape by changing the
bend away from the light. amount of water in them resulting in folding up and drooping
of leaves. This movement is independent of growth.
5.
Plants respond to a stimulus by growing in a particular direction
and the movements due to growth. This growth is directional.
Movement of shoots towards light indicates phototropism, i.e.
Neuron cell body movement occurs in response to light.
Nucleus
9. (a) (i) Abscisic acid
Axon Dendrite • inhibits growth
Myelin
• causes dormancy of seeds, wilting of leaves
sheath
• causes stomatal closure
(ii) Cytokinin
6. (i) Reflex actions. • promotes cell division

(ii) Reflex actions. • delays aging in leaves


• reduces apical dominance
(iii) Reflex actions.
(b) Chemotrophic movement is the growth of plant in
(iv) Voluntary actions.
response to a chemical stimulus, e.g growth of pollen
(v) Reflex actions. tube towards ovules during fertilisation.
14 Chapter 6 : Control and Coordination

10. (a) Pituitary gland (master gland) is present at the base of On reaching the axonal end, it causes the nerve endings
brain. to release a chemical, which diffuses across a synapse and
(b) Thyroid gland is present just below the neck. stimulates the presynaptic membranes of next neuron. In this
way, the electrical signal reaches the brain or spinal cord.
(c) Pancreas is found just below the stomach.
The response from brain (or spinal cord) is similarly passed
(d) Adrenal glands are present on top of each kidney.
on to the effector, that undergoes the desired response.
Pineal Gland 13. Human brain is main coordinating centre of the body, which
Hypothalamus enables an organism to think and take decisions.
Pituitary Gland

Parathyroid Thyroid Gland


Glands
Thymus

Pancreas Adrenal Glands




Testes Functions of different parts of the brain are:


Medulla oblongata: It controls involuntary actions and
regulates reflex responses. It also controls blood pressure,
11. Hypersecretion (more secretion) or hyposecretion (less
salivation and vomiting.
secretion) of different hormones lead to various disorders in
our body. The three common examples are: Cerebellum: It controls and coordinates different muscular
actions. It is responsible for voluntary actions and maintains
(i) Goitre: lodine acts as the necessary component for the equilibrium of the body during walking, drinking, catching,
synthesis of thyroxine hormone from thyroid gland. This etc.
disorder is caused due to the deficiency of iodine that
Forebrain: It has following parts:
leads to hyposecretion of thyroxine.
Cerebrum: performs thinking, reasoning, speech, intelligence
(ii) Gigantism and dwarfism: Hypersecretion of growth and usage of information.
hormone results in gigantism (very tall individual).
Olfactory: lobes are responsible for detecting smell from
On the contrary, the hyposecretion or deficiency of different receptors.
growth hormone at an early stage of life makes the Hypothalamus: controls body temperature, urge of eating,
person very short, i.e. causes dwarfism. drinking, etc
(iii) Diabetes mellitus: Insulin secreted by pancreas helps 14. Adrenal gland secretes adrenaline and corticoid hormones.
to lower the blood glucose level. When it is secreted in Adrenaline increases the heart beat and breathing rate which
less amount, the body suffers from diabetes. More and results in the supply of more oxygen to muscles. It reduces
more glucose gets accumulated in the body. the blood to the digestive system and skin; as a result the
blood further reaches the skeletal muscles. All these responses
12. All the information from the environment is detected by the
together prepare the body to deal with the emergency
receptors (sense organ) present in The body. The stimulus situations.
received by the receptor is passed on in the form of electrical
15. (a) Hormones are the chemical substances, which control
signals to dendrites and then to cyton of the neuron. The
and coordinate the activities performed by organisms.
impulse then travels along the axon of the neuron.
Characteristics of Hormones
(i) They are poured directly into the blood stream
and are carried throughout the body by circulatory
system.
(ii) They act only on the specific target organs.
(b) (i) Testosterone produced by testes regulates the
changes in male during adolescence period.
(ii) Insulin (decrease blood sugar) and glucagon
(increase blood sugar), secreted by pancreas
coordinates the sugar level in blood.
16. No reflex action is not only function of spinal cord; it is function
of central nervous system which consists of spinal cord and
brain.


15 Chapter 6 : Control and Coordination BIOLOGY 15

17. Geotropism is necessary as it directs roots to move down the transmission in the body. The small molecules secretive by the
soil to collect minerals and water for plant growth. endocrine gland are needed to be diffused with target organs.
18. Adrenal gland So the chemicals produced by endocrine glands are usually in
the form of small molecules so that diffusion occurs faster.
19. Auxin is given in an increased dose which promotes the
development of seedless fruits. Other functions of auxin are: it (ii) The five endocrine glands found in the human body are:
is responsible for elongation of cells at the shoot tip and also is • Pituitary gland which secretes growth hormone.
responsible for showing positive phototropism. • Thyroid gland which secretes thyroxine hormone.
20. (i) The endocrine system is responsible for all the chemical • Parathyroid gland which secretes parathormone.

NTSE & OLYMPIAD QUESTIONS

1. (c) The nucleus is present in the cyton part of the nerve cell. 10. (c) An action potential is a “spike” of electrical discharge
that travels along the membrane of a cell.
2. (a) Sensory receptor, in physiology, any structure which, on
receiving environmental stimuli, produces an informative 11. (a) Chemical synapses are specialized junctions through
nerve impulse. which the cells of the nervous system signal to each
other and to nonneuronal cells such as those in muscles
3. (d) Receptor → spinal cord → muscle or glands. Chemical synapses allow the neurons of the
4. (d) Nervous and MuscularThe two tissues that provide central nervous system to form interconnected neural
control and coordination in multi-cellular animals are circuits.
nervous tissue and endocrine tissue. 12. (b) Neurons are electrically excitable cells in the nervous
The major function of endocrine tissue is chemical

system that process and transmit information. In
coordination. These tissues secrete hormones which vertebrate animals, neurons are the core components of
control many functions in our body. the brain, spinal cord and peripheral nerves.
Some times they even control the emotions of the 13. (d) A reflex arc is the neural pathway that mediates a reflex
action. In higher animals, most sensory neurons do not


organisms. The nervous tissue contains sensory points


like synapse and motor nerve points which bring pass directly into the brain, but synapse in the spinal
cord. This characteristic allows reflex actions to occur
responses and they coordinate with the brain.
relatively quickly by activating spinal motor neurons
Hence, Nervous and endocrinal tissues provide control

without the delay of routing signals through the brain,
and coordination in animals. although the brain will receive sensory input while the
5. (a) Cranial and spinal nerves are part of peripheral nervous reflex action occurs.
system. The 12 pairs of cranial nerves are part of peripheral 14. (a) The peripheral nervous system is part of the nervous
nervous system and are mostly concentred with head, system, and consists of the nerves and neurons that
neck and facial region of the body (vagus nerve X being reside or extend outside the central nervous system (the
the part of autonomous division serves as an exception). brain and spinal cord) to serve the limbs and organs, for
These are namely olfactory, optic, vestibulocochlear, example.
vagus, oculomotor, trochlear, abducens, spinal accessory, 15. (a) The prosencephalon (fore brain), the mesencephalon
trigeminal, facial, glossopharyngeal and hypoglossal (midbrain), and rhombencephalon (hindbrain) are
nerves. Spinal nerves are attached to spinal cord. There the three primary portions of the brain during early
are total 31 spinal nerves which are grouped as cervical development of the central nervous system. Fore-brain
(C1 to C8), thoracic (T1 to T12), lumber (L1 to L5), sacral is the main thinking part of brain.
(S1 to S5), and coccygeal nerves (C1). 16. (c) Dendrite → cell body → axon → axonal end.
6. (b) Neurotransmitters and hormones 17. (d) Receptors → sensory neuron → spinal cord → motor
7. (a) Neuron is the basic unit of nervous system neuron → muscle.
8. (d) Vasopressin is a neurohypophysial hormone found 18. (d) It controls autonomic functions and relays nerve
in most mammals. It’s two primary functions are to signals between the brain and spinal cord.The Medulla
retain water in the body and to constrict blood vessels. oblongata is responsible for controlling several major
Vasopressin regulates the body’s retention of water by autonomic functions of the body : respiration (via dorsal
acting to increase water reabsorption in the kidney’s respiratory group and ventral respiratory group), blood
collecting ducts, the tubules which receive the very dilute pressure, heart rate, reflex arcs and vomiting.
urine produced by the functional unit of the kidney, the 19. (a) The skull is a bony structure found in many animals
nephrons. which serves as the general framework for the head.
It plays a key role in homeostasis, by the regulation of Those animals having skulls are called Craniates. The
skull supports the structures of the face and protects the


water, glucose and salts in the blood.


head against injury. The skull is made up of two bones:
9. (c) Control and co-ordination in an organism is provided by the cranium and the mandible.
the nervous system. The nervous system consists of the
brain and the spinal cord and the nerves.
16 Chapter 6 : Control and Coordination BIOLOGY 16

20. (b) The cerebellum is a region of the brain that plays an is an endocrine gland about the size of a pea that sits in
important role in the integration of sensory perception a small, bony cavity covered by a dual fold at the base of
and motor output. Many neural pathways link the brain.
the cerebellum with the motor cortex which sends 30. (d) Seeds release CO2 during respiration, which is absorbed
information to the muscles causing them to move and by KOH creating a partial vacuum in the flask. To fill that
the spinocerebellar tract which provides feedback on the water rises.
position of the body in space (proprioception).
31. (b) A - (q); B -(p); C - (s); D - (u); E - (t); F - (r)
21. (c) Thyroxine is critical to the regulation of metabolism
32. (d) A - (s); B - (p); C - (r); D - (q)
and growth throughout the animal kingdom. Calcium
is required for the thyroid gland to make thyroxine 33. (d) A - (r); B - (s); C - (p); D - (q)
hormone. 34. (c) Both (i) and (ii)
22. (c) Statements (ii) and (iii) are wrong in their sense because 35. (c) Both (i) and (ii)
sensory neurons carry signals from receptors to spinal
36. (a) Stems are negatively geotrophic while roots are positively
cord and motor neurons carry signals from spinal cord to
geotrophic.
the muscles.
37. (d) Neither (i) nor (ii)
23. (c) The forebrain comprises the majority of the brain in
humans. It is the main thinking part of the brain. It 38. (c) Both (i) and (ii)
controls most bodily functions and behaviour. 39. (c) Both (i) and (ii)
The forebrain includes the cerebrum, thalamus, 40. (d)
hypothalamus and limbic system. Visual
41. (d)
perception,language, smell, memory, hearing, sight, etc.,
are the functions of the forebrain. Involuntary actions 42. (d)
like salivation, vomiting, blood pressure are controlled 43. (a)
by the medulla in the hindbrain. Cerebellum maintains 44. (a)
balance and posture of the body.
45. (a)
24. (a) In mammals, the adrenal gland also known as suprarenal
46. (d)
glands are the triangle-shaped endocrine glands that sit
on top of the kidneys; their name indicates that position. 47. (a) Synapse is the points of contact between neurons where
information is passed from one neuron to the next.
25. (c) The pancreas is a gland organ in the digestive and
endocrine systems of vertebrates. It is both exocrine 48. (b) Reflex action is a voluntary, unplanned sequence or
(secreting pancreatic juice containing digestive enzymes) action and nearly instantaneous movement in response
and endocrine (producing several important hormones, to a stimulus
including insulin, glucagon, and somatostatin). 49. (b) Chemotropism is the growth of organisms navigated by
26. (b) Testosterone is a steroid hormone from the androgen chemical stimulus from outside of the organism
group. Testosterone is primarily secreted in the testes 50. (d) Goitre is a swelling of the thyroid gland that causes a
of males. It is the principal male sex hormone and lump in front of the neck.
an anabolic steroid. Estrogens are a group of steroid
51. (d) Lymph
compounds, named for their importance in the estrous
(i) It is the structural and functional unit of lungs.
cycle, and functioning as the primary female sex
(ii) It is thin walled, has a large surface area and is richly
hormone.
supplied with blood vessels.
27. (a) Adrenaline : Pituitary gland
52. (a) Neuron
(i) It is the structural and functional unit of kidneys.
28. (c) (ii) It removes nitrogenous wastes from the blood.
53. (d) Auxin
29. (a) The hypothalamus links the nervous system to (i) It promotes cell elongation.
the endocrine system via the pituitary gland.The (ii) It can cause formation of seedless fruits.
hypothalamus, is located below the thalamus, just above
54. (d) Respiratory system
the brain stem. The pineal gland is a small endocrine
(i) This system has direct connection and control over
gland in the brain. It is shaped like a tiny pine cone, and
the tissues or organs.
is located near the center of the brain, between the two
(ii) The information is transmitted very fast and the
hemispheres, tucked in a groove where the two rounded
effect is short lived.
thalamic bodies join. The pituitary gland, or hypophysis,

❑❑❑
BIOLOGY 17

7 ANSWERS & SOLUTIONS


MULTIPLE CHOICE QUESTIONS

1. (c) Variations are not responsible for maintenance of body 20. (b) The transfer of pollen grains from anther to stigma of a
design features. pistil is called pollination.
2. (d) Characters are transmitted from parents to offsprings 21. (b) The period of life, when the body undergoes changes,
through genes. leading to reproductive maturity, is called adolescence.
Adolescence begins around the age of 11 and lasts upto
3. (b) The number of chromosomes in parents and offsprings 18 or 19 years of age.
of a particular species remains constant due to halving of
22. (d) (iii) and (iv)
chromosomes during gamete formation.
23. (c) Chromosomes get halved during gamete formation
4. (d) Bryophyllum reproduces by the buds present in their
24. (c) (i), (ii) and (iv)
notches along the leaf margin of Bryophyllum which falls
on the soil and develops into new plants. 25. (a) Fertilization
5. (d) Yeast. It show reproduction by budding. 26. (b) The two oviducts in a human female unite into an elastic
bag like structure known as Uterus.
6. (b) Yeast fungus. Amoeba, Paramaecuim, Plasmodium,
27. (c) The correct answer is Fallopian tube.
Hydra spyrogyra are not produced by spore formation.
28. (d) The process of release of eggs from the ovary is called
7. (b) Division of a cell into many cells
ovulation
8. (d) (i) and (iv)
29. (b) Hepatitis is a water borne viral disease and not a sexually
9. (d) The correct answer is Regeneration. It is a process in transmitted disease.
which the organism can be cut into any number of pieces 30. (a) It is because condoms are a barrier method of protection
and each piece grows into a complete organism. which doesn’t even allow sperms to enter the vagina. On
10. (a) Stock is the portion on which grafting is done and it the other hand, spermicides kill the sperms as soon as
provides the roots. they enter vagina hence prevent fertilization. Vasectomy
is a surgical method that cuts vasa deferentia thus doesn’t
11. (b) Hydra even allow the sperms to get ejaculated in the urethra.
12. (a) Does not produce new variety. Leads to overcrowding Thus, only coil mechanism prevent implantation.
around the parent plant. Very little possibility of dispersal. 31. (b) A condom is the kind of contraception that protects a
13. (b) During favourable conditions amoeba reproduces by person from acquiring a sexually transmitted disease up
Binary fission . The division or splitting of a cell into two to some extent.
equal parts is called binary fission. It takes place under 32. (d) Syphilis disease is transmitted sexually.
favourable condition 33. (b) The IUCD is a very effective form of emergency
14. (a) In unicellular organisms like bacteria, unicellular algae, contraception which is effective for up to five days
or Amoeba, reproduction is similar to growth, i.e., an after you have had unprotected sex (intercourse). It
prevents more than 9 out of 10 pregnancies which would
increase in the number of cells. As Amoeba, Spirogyra,
otherwise have occurred.
and Yeast all reproduce by asexual means
34. (d) AIDS is a deadly disease which is caused by HIV virus.
15. (b) Germination is a process occurring in plants in which
the embryo develops into a seedling under appropriate 35. (b) Growing foetus derives nutrition from mother through
placenta
condition.
36. (b) The average age for girls to begin puberty is 10-12
16. (d) Glands like prostate and seminal vesicles add their

1
secretions to vas deferens so sperms are easily transported
and nurtured till maturation. MARK QUESTIONS

17. (a) Sperms formed in testis are delivered through the vas
deferens which joins with another tube called urethra 1. The term used to describe the male and female gametes fusion
coming from the urinary bladder. is fertilization. Fertilization can be described as the fusion of
18. (c) Placenta is embedded in the uterine wall. the male gametes (pollen) with the female gametes (ovum)
19. (d) Reproduction is not essential for maintaining the life of to form a diploid zygote. It is a process that occurs after the
an individual. pollination of the carpel.
18 Chapter 7 : How Do Organisms Reproduce

2. In flowering plants, two male gametes from a single pollen 6. Reproduction helps in providing stability to populations
grain fuse with two female gametes, the egg and central cells, of species by ensuring the continuation of the species.
to form the embryo and endosperm, respectively. Reproduction ensures that there are always new individuals
being produced to replace those that die, and that the genetic
3. The pollen tube ultimately enters an ovule through the
diversity of the species is maintained.
micropyle and penetrates one of the sterile cells on either side
7. If fertilization does not occur, the uterus undergoes the process
of the egg (synergids). These synergids begin to degenerate
of menstruation, which involves shedding of the lining of
immediately after pollination. the uterus, along with blood and other materials, which are
4. The length of the menstrual cycle varies from woman to expelled from the body through the vagina.
woman, but the average is to have periods every 28 days. 8. Along the path of vas deferens, the sperm is mixed with
Regular cycles that are longer or shorter than this, from 21 to fluids from the seminal vesicles, the prostate gland, and the
40 days, are normal bulbourethral gland. These fluids provide nourishment, buffer
against the acidic environment of the female reproductive
5. After the process of fertilization, the ovary swells up and
tract, and help in the movement and viability of the sperm.
develops into a fruit. A fruit develops from the maturation of
9. The above situation can be explained by the phenomenon
one or more flowers, and the ovary of the flower forms all or
of self-pollination. In a bisexual flower, the pollen from the
part of the fruit.
stamens can reach the stigma of the same flower or another
6. Artificial method of vegetative propagation by stem cuttings flower on the same plant, leading to fertilization and fruit
and bud grafting. production, even if the young stamens are removed. Self-
pollination can occur due to various factors, such as wind,
7. There is no genetic variation, such plants are less adaptable to
gravity, or insect activity.
environmental changes.
8. Sexual reproduction is the method of reproduction involving
fusion of gametes from both the parents, i.e. ovum (female) 3 MARKS QUESTIONS

and sperm (male).


1. Complex organisms are not simply a random collection of
9. During pollination, pollen grain falls over stigma. This pollen cells where specialized cells are organised as tissues, and
grain absorbs water and nutrients and produces a tube called tissues ale organised into organs which then have to be
pollen tube. placed at definite positions in the body. In such a carefully
organised situation, it is not easy to develop new individual
10. The fusion of sperm with the egg is called fertilisation. This through regeneration.
leads to thie formation of zygote, i.e. the product.
2. (i) No, there is no relationship between size of organism and

2


its chromosome number.


MARKS QUESTIONS
(ii) No, the process of reproduction follows a common


1. Seminal vesicles and the prostate gland add their secretions pattern and is not dependent on the number of
chromosomes.
so that the sperms are in a fluid (semen) which makes their
transport easier and this fluid also provides nutrition. (iii) Yes, since the major component of chromosome is DNA.


2. When vas deferens in males are blocked surgically, sperm If there are more chromosomes in a cell, the quantity of
DNA will also be more.
transfer is be prevented. Similarly, when Fallopian tubes are
blocked in females the egg will not be able to reach the uterus 3. (i) The plants that cannot produce viable seeds such as


thereby preventing pregnancy. banana, seedless g rapes and oranges, etc., can be easily
3. Sexual reproduction leads to new combination Of genes as it grown by vegetative propagation.
involves two parents and meiosis. This produces variation in
(ii) It is an easier, less expensive and a rapid method of
offspring. Variations are the basis for evolution.


propagation.
4. The human male gamete, i.e., the sperm, is small, motile,
(iii) Genetically identical plants can be produced.
and has a tail for movement, while the female gamete, i.e.,


the egg, is larger, non-motile, and contains nutrients for the 4. The reasons for adopting contraceptive method are as
developing embryo. follow:
5. The embryo gets nourishment inside the mother’s body through (i) To control the birth rate and prevent the increase in


the placenta, which is a specialized organ that connects the population.


developing foetus to the mother’s uterus. The placenta allows
(ii) To reduce the adverse effects on mother’s body due to
for the exchange of nutrients, gases, and wastes between the


frequent pregnancy.
mother and the foetus through the umbilical cord.
(iii) To provide safety from sexually transmitted diseases.

BIOLOGY 19

5. Binary fission is a type of asexual reproduction in which a single Advantages of vegetative propagation: It produces
parent cell divides into two genetically identical daughter cells. genetically identical plants, which ensures the retention
This process occurs in unicellular organisms such as bacteria. of desirable traits in the offspring. It allows for the rapid
For example, in the bacterium E. coli, the DNA replicates and propagation of plants without the need for seeds, which
the cell divides into two identical daughter cells. can be advantageous in situations where seed production is
difficult or impractical.
6. Oviparous animals lay eggs outside the body, and the embryos
develop outside the mother’s body. Examples birds, reptiles,
and many species of fish. Viviparous animals give birth to
live young that develop inside the mother’s body. Examples
mammals such as humans, dogs, and whales.
5 MARKS QUESTIONS

1. If the ovum (or egg) does not get fertilised (due to non-
7. The female reproductive system consists of the ovaries,
availability of sperm in the female body) then the thick and
fallopian tubes, uterus, cervix, and vagina. The ovaries
soft inner lining of uterus is no longer needed and hence it
produce and release eggs, which are then transported to the
breaks. So, the thick and soft inner lining of uterus alongwith
fallopian tubes. Fertilization occurs in the fallopian tubes, and the blood vessels and the dead ovum (or egg) comes out of the
the fertilized egg then travels to the uterus, where it implants vagina in the form of blood called menstruation. Menstruation
and develops into a foetus. occurs after the interval of every 28 days and the time period
8. Self-pollination occurs when pollen from the same flower between ovulation and menstruation is about 14 days.
pollinates the stigma of that flower. Cross-pollination occurs 2. (a) Female Reproductive System
when pollen from one flower fertilizes the stigma of a different (i) Ovary
flower. Cross-pollination can result in greater genetic diversity (ii) Fallopian tube or oviduct
in the offspring. (iii) Uterus
Uterus Fallopian tube
9. The advantages of sexual reproduction over asexual
reproduction are:
• Sexual reproduction leads to greater genetic diversity,
which allows for better adaptation to changing Ovary Ovary
environments. Endometrium
• Sexual reproduction helps in the elimination of deleterious Vagina
Cervix
mutations through the process of recombination.
(b) When the human egg is not fertilized, it passes out of
• Sexual reproduction can produce offspring with stronger
the body along with the lining of the uterus during the
immune systems, as they inherit a mixture of immune
process of menstruation. Menstruation is a regular,
system genes from both parents.
natural process that occurs in females during their
10. (a) reproductive years (Age 15 – 50). It is the shedding of
the uterine lining, which occurs approximately once a
(i) Implantation is the process by which the blastocyst, a
month. The menstrual cycle is controlled by hormones
ball of cells that develops after fertilization, attaches itself
that are produced by the hypothalamus, pituitary gland,
to the lining of the uterus and begins to grow into an
and ovaries. The cycle is divided into four phases: the
embryo.
menstrual phase, the Follicular phase, the ovulatory
(ii) Placenta is a specialized organ that connects the phase, and the Luteal phase. The menstrual phase is the
developing fetus to the mother’s uterus and allows for first phase of the cycle and occurs when the uterine lining
the exchange of nutrients, gases, and wastes between the sheds. This results in bleeding, which typically lasts for
mother and the fetus through the umbilical cord. 3-5 days. The blood is a mixture of blood, mucus, and
uterine tissue.
(b) The average duration of human pregnancy is about 280
days or 40 weeks from the first day of the last menstrual 3. (a) (i) Stamen
period. (ii) Pollen
11. Vegetative propagation is a method of asexual reproduction (iii) Pistil
in plants that involves the production of new plants from (iv) Ovule 

vegetative parts such as roots, stems, and leaves, without the petal
involvement of gametes or seeds. pollen
anther stigma
Two examples of vegetative propagation are: Runners style carpel
stamen
or stolons in strawberry plants, which produce new plants at filament ovary
the nodes along the runner. Tuber or rhizome in potato plants, sepal ovule
which can be cut into pieces, each containing an “eye” or receptacle stem
bud, and planted to produce new plants.
20 Chapter 7 : How Do Organisms Reproduce

(b) The function of the flower in the flowering plant is to (iii) The main difference between fission and fragmentation
produce reproductive structures that contain the male is in the way the parent organism divides and forms new
and female gametes, and facilitate their fusion to form individuals. Fission is a type of asexual reproduction
seeds, which can develop into new plants. in which the parent organism divides into two or more
(c) Fertilization in plants daughter individuals, each of which has the potential
to grow into a new individual. This division occurs
4. (i) Four asexual modes of reproduction are binary fission,
through mitosis, which is the process of cell division in
budding, spore formation, and vegetative propagation.
which the parent cell divides into two identical daughter
(ii) Amoeba reproduces asexually by the process of binary cells. Fission can occur in different ways, such as binary
fission. In this process, the cell divides into two daughter fission, where the parent cell divides into two identical
cells, each of which has the same genetic material as daughter cells, or multiple fission, where the parent cell
the parent cell. The steps involved in the process are: divides into several daughter cells.
The amoeba cell elongates and duplicates its genetic
Fragmentation, on the other hand, is a type of asexual
material. The cell then constricts at its center, and the
reproduction in which the parent organism breaks up
two daughter cells are formed. Each daughter cell then
into two or more fragments, and each fragment has the
grows and develops into an adult amoeba.
potential to grow into a new individual. The process of


fragmentation can occur naturally in some organisms


or can be induced by external factors, such as physical
Nuceous divide damage. Each fragment can regenerate the missing body
Parent Cytoplasm
Cell divide parts and develop into a new individual.

Two daughter cells




HOTS QUESTIONS

1. DNA copying in reproduction is important for maintenance to form a small group of cells or callus
of body designs and features. Moreover, DNA copying 3. In binary fission, the parent cell divides itself into two equal
leads to variations. Variation is useful for the survival of and identical daughter cells. It is the most common form of
species. reproduction in prokaryotes such as bacteria. In multiple
fission, a single parent cell is divided into many daughter cells.
2. In tissue culture, new plants are grown by removing tissue or
separating cells from the growing tip of a plant. The cells are 4. Gamete cell are the reproductive cells of an organism. Female
then placed in an artificial medium where they divide rapidly gametes are referred to ova or egg cells and male gametes are
known as sperms.

CASE BASED QUESTIONS

I. (1) (d) In barrier methods of preventing pregnancy, the


 II. (6) (b) Intra-uterine device is placed inside the uterus by a


physical devices such as condom and diaphragm doctor or a trained nurse. Copper T is a common
are used. Condoms are rubber tubes used by males intra-uterine device.
whereas diaphragm are rubber cups used by females.
(7) (b) Copper
(2) (c) Physical barriers prevent the sperm from meeting the


ovum by acting as a barrier between them. (8) (c) It suppresses fertilising capacity of sperm

(3) (d) Chemical methods of birth control include oral



(9) (a) Intra-uterine devices are highly effective in preventing


pills and vaginal pills. Oral pills are combination of unwanted pregnancies. But they come with one
estrogen and progesterone which prevent ovulation disadvantage, that is they can get expelled anytime
without the knowledge of women. Couple continue
(release of egg during monthly cycle), so they prevent
active sexual life thinking that their birth control
fertilisation. Vaginal pills are inserted in vagina
device is still in action.
before intercourse and release spermicides which kill
sperms. (10) (d) Intra-uterine devices do not protect against sexually


transmitted diseases. Periods may become heavier,


(4) (d) Both (b) and (c)
longer and more painful and there are chances of
(5) (d) All of these pelvic infection.
BIOLOGY 21

III. (11) (b) Frog Pigeon Elephant (19) (d) After implantation, a disc like tissue develops


between thick uterine wall and embryo, called


(12) (b) X could be fish or amphibian whereas Y could
placenta. Placenta links the embryo to the mother


be bird or reptile. Animals like fish or amphibian


through umbilical cord. All the requirements of the
produce large number of eggs and sperms which
developing foetus like nutrition, respiration, excretion
fuse in external medium like water. Animals like
etc., are met from mother’s body through placenta.
birds or reptiles produce few eggs at a time. They lay
This is because in placenta, embryo’s blood vessels
fertilised eggs which are protected by outermost hard
lie in close association with mother’s blood vessels.
calcareous shell.
(20) (d) All of these.
(13) (b) Development of zygote takes place outside the body
V.


(21) (b) X could be Syphilis, Y could be AIDS and Z could be


of Y inside the laid egg whereas zygote develops into


gonorrhoea.
complete baby inside the body of female Z.
(22) (c) Human immunodeficiency virus (HIV) cause immuno
(14) (c) Cat, dog and human undergo internal fertilisation so


deficiency syndrome, a condition characterised by




their eggs are not covered by calcareous shell or jelly


progressive failure of immune system allowing life
covering.
threatening conditions.
(15) (a) To keep the eggs moist and offer some protection


(23) (d) All of these


from predators
(24) (d) None of these.
IV. (16) (d) All of these
(25) (b) Sterilised needles are free from any kinds of germs.
(17) (b) First of all ovulation takes place. Then the eggs get


VI. (26) (a) Testosterone


fertilised by sperm. This leads to formation of zygote.
(27) (d) Fluid helps to bind the sperms together
Zygote divides to form few celled embryo which gets
embedded at the proper site in the thick lining of (28) (b) Sperms formation requires lower temperature than


uterus. This is called implantation. body temperature

(18) (c) Zygote is formed by fusion of sperm and egg i.e.,



(29) (d) Sperms formation requires 1–3°C higher temperature


fertilisation. It is single celled and it gives rise to than normal body temperature.
embryo by repeated cell divisions. (30) (c) Slightly basic

NCERT QUESTIONS

1. Variations allow organisms to exist in diverse habitats or niches. the form of DNA, which is the information source of making
In its absence, a species may remain restricted to a particular proteins. Therefore, a basic event in reproduction is creation
area. If this area gets drastically altered due to various natural of DNA copy for the next generation.
or man-made causes, the species may be wiped out. However, 5. Differences between pollination and fertilisation are:
if some variations are present in few individuals, it would help
Pollination
them to colonise other habitats and survive. But, if variations
are present in a single organism, there would be a very little The transfer of pollen grains from anther to stigma of a female
chance for it to survive. flower is called pollination.
2. Spores are tiny, spherical, asexual reproductive bodies. They It involves only the male gamete (pollen grain).
are covered with a hard protective wall. This enables them to Fertilisation
survive in unfavourable conditions. They can only germinate
The fusion of male and female gamete to form zygote is called
and produce new plant under favourable conditions thereby
fertilisation.
benefitting the organism from dying out.
It involves both male and female gamete.
3. Vegetative propagation is practiced for growing some types of
plants because of the following reasons: 6. Changes seen in girls at the time of puberty are as follows:
(i) Plants that have lost their capability to produce seeds (i) Growth of hairs in armpits and pubic region.
can be propagated by this method. (ii) Mammary glands (breast) develop and hips broaden.
(ii) It helps to grow plants bearing superior traits, as they are (iii) Uterus, vagina, Fallopian tube enlarge and pelvis widens.
genetically identical to the parent plant.
(iv) Menstruation and ovulation also start.
(iii) It is used for growing plants which require a longer time
to grow and become mature. 7. Functions of testes in human beings include.

4. Chromosomes in the nucleus of a cell contain information for (i) production of male sex cells-sperms.
inheritance of features from parents to the next generation in (ii) production of male sex hormone-testosterone
22 Chapter 7 : How Do Organisms Reproduce

8. 9. The different methods of contraception are:


(i) Barrier Methods The physical devices such as
condoms and diaphragm are used. These are physical
barriers, which prevents sperms from meeting the ova.
Anther (ii) Hormonal Methods These contain hormonal
Stigma Stamen preparations in the form of ‘pills’ which prevent the
(Androecium) release of ovum.
Petal
(Corolla) Carpel (iii) Chemical Methods The vaginal pills contain the
(Gynoecium) chemicals called spermicides which kill the sperms.
Ovary
Sepal (iv) Surgical Methods In males, a small portion of sperm
(Calyx) duct is cut by surgical method. The cut end is tied
Receptacle properly to prevent the sperms from coming out. In
Flower
stem females, a small part of the Fallopian tube/oviduct is cut
and tied to prevent the egg from entering the oviduct.
Flowers are the reproductive unit of a plant. They are not only
involved in reproduction, but are also a source of food for 10. Unicellular organisms are made up of only a single cell, in which
other living organisms. They are a rich source of nectar. all the functions are performed by single cell. So, reproduction
• The different part of a flower is labelled below. is done by simple asexual methods but in multicellular
• Pedicel: It is the stalk of a flower. organisms various cells perform different functions. So,
• Calyx or Sepals: The outermost covering or whorl of a production of all these specialised cells is required and simple
flower is called calyx and its units are called sepals.
methods for reproduction are insignificant. Thus, reproduce
• Corolla or Petals: Next to the calyx is present a corolla
which is a colourful whorl and helps in attracting by sexual reproduction
pollinating insects. The units of the corolla are called 11. A species occupies a well-defined niche in an ecosystem, using
petals. its ability to reproduce. During reduction, copies of DNA pass
• Stamen: It forms the male reproductive part of the flower.
from generation to the next. This copying of DNA takes place
Each stamen has a long filament and a terminal anther.
The anther contains pollen grains. with consistency in reproducing organisms, this is important
• Carpel or Pistil: It forms the female reproductive part of for the maintenance of body design features (physiological
the flower. Each carpel or pistil consists of three parts as well as structural) which allows the organism to use that
which are stigma, style and a basal swollen part called particular niche. Reproduction is therefore, linked to the
the ovary. stability of population of a species.

CBSE PAST YEAR QUESTIONS

1. The embryo gets nutrition from the mother’s blood with the with the new population and thus ensures the survival of the
help of a special tissue called placenta. This is a disc-like species. In the absence of reproduction one particular species
tissue which develops between the uterine wall and embryo. will disappear with time.
As mother eats, the food passes through the digestive system 7. (i) The process depicts binary fission, a method of asexual
where the body breaks it down into small particles. These reproduction
nutrients travel through the mother’s blood stream and get
(ii) Binary fission occurs in Amoeba, Euglena and
exchanged with the blood stream of foetus through placenta.
Paramecium, etc.
2. No, copper-T does not prevent the transmission of sexually
transmitted diseases. Copper-T only prevents implantation. (iii) Differences between fission and fragmentation are:
The only safe method that can be used to prevent the Fission
transmission of sexually transmitted diseases is condoms.
It is the division of parent body into two identical daughter
3. Sexual reproduction is the method of reproduction involving cells.
fusion of gametes from both the parents, i.e. ovum (female)
It occurs in unicellular organisms or multicellular organisms
and sperm (male).
with simple body organisation.
4. Ovary in females is responsible for the production of female
e.g. Amoeba, Plasmodium.
gamete (ova) and also produces hormones, i.e. estrogen and
progesterone. Fragmentation
5. When vas deferens in males are blocked surgically, sperm It is the division of parent body into two or more small
transfer is be prevented. Similarly, when Fallopian tubes are fragments.
blocked in females the egg will not be able to reach the uterus
It occurs only in multicellular organisms with complex cellular
thereby preventing pregnancy.
organisations.
6. Reproduction is directly linked to the stability of the population
e.g. Spirogyra.
of species because it helps in replacing the ageging population
BIOLOGY 23

8. (i) Budding as seen in Hydra is a type of asexual reproduction 13.


which involves only a single parent. In this, a daughter
Part Function
organism is formed from a small projection called bud
and it later detaches to grow into a new independent A-Anther Formation of pollen
individual. B-Style Lifting stigma to receive pollen
(ii) In budding, the new individuals develop from the parent C-Ovary Contains ovule which develop into seeds
as an extra outgrowth whereas in fission, parent’s body while ovary forms the fruit
divides into identical daughter cells.
9. regeneration is the ability of some organisms to give rise to 14. (a) The process by which a new plant grows and develops
new organisms when the individual is cut or broken up into from a seed into a seedling under appropriate conditions
many pieces. It is seen in Hydra and Planaria. is called germination.

Reproduction is the process by which a living organism is able During germination various parts of seeds are formed
to produce new individuals of its own kind. that protect the growing embryo like seed coat.

Regeneration in multicellular organism like Hydra (b) The part labelled incorrectly are:
(i) Male germ cells - Should be pollen grain
(i) It is carried out by specialised cells.
(ii) Pollen grain - Male germ cells
(ii) When Hydra is cut or broken up into many pieces these
specialised cells proliferate and make large number of (iii) Ovule - Should be the ovary
cells. 15. (a) The two major reasons for the appearance of variation
(iii) From this mass of cells, different cells undergo changes to among the progeny formed by sexual reproduction are
become various cell types and tissues. (i) Crossing over is the process in which DNA is
exchanged between the homologous chromosomes
(iv) These changes take place in an organised sequence during meiosis, This causes variation in the progeny.
referred to as development thereby making each piece (ii) The progeny formed due to sexual reproduction has
to grow into a separate individual. characteristics from the mother as well as the father.
10. (i) In slice 1, no change will be observed or it will remain sterile This lead to result in variation among the progeny.
because it lacks moisture.
(b) (i) Pollen grain
(ii) A white cottony mass surrounded with black pin (ii) Stigma attract the pollen grains from the air with its
head-like structures are seen spreading on the surface
sticky tip for reproduction to take place.
of slice 2. This is because tiny spores of Rhizopus are
always present in air which thrive in humid conditions. (iii) Pollen tube, it deliver sperm cells to the female
This is why on the slice 2 kept in moist and dark place, gametophyte for double fertilization.
formation of sporangia and spores takes place, which are (iv) Female gamate that forms zygote after fertilisation.
favourable for the growth of fungus. The ovule becomes a seed after fertilization, while
(i) In slice 3, no change is observed (remains sterile) as it is the ovary becomes a fruit.
kept at low temperature in the refrigerator.
16. Pollination is the process of the transfer of pollen grains from
11. The main difference between sexual and asexual reproduction anther to the stigma of the flower.
involves the production and fertilisation of gametes in
There are two modes of pollination in flowering plants:
sexual reproduction which do not occur in asexual mode of
reproduction. (i) Self-pollination
Sexual reproduction is considered to be superior over (ii) Cross-pollination
asexual reproduction as it leads to variations, while asexual Self-pollination
reproduction does not induce variations among progeny
individuals. In this, the pollen grain from the stamen of a flower is
transferred to the stigma of the same flower.
Advantages of variations in individuals are
Cross-pollination
(i) It helps in the survival of species.
In this, the pollen grain from the stamen of a flower is
(ii) It is the basis of evolution.
transferred to the stigma of different flower.
(iii) It brings adaptation in individuals.
17. A bisexual flower has the male as well as female reproductive
12. Characteristics of sexual reproduction are: organs. If the young stamen (i.e. male unit) is removed
(i) It takes place with the help of gametes. artificially, the flower still has its pistil (i.e. female unit) intact.
(ii) It is biparental type of reproduction. Therefore, cross-pollination can occur. When the pollen
(iii) It is a comparatively slower method of multiplication. grains from the anther of another flower are transferred to
the stigma of this flower with the help of pollinating agents as
(iv) Abundant variations occur during this method.
insects, bees, wind and water, it causes cross-pollination. After
(v) It is important for evolution. the pollen grains fall on stigma, the next step is fertilisation,
(vi) It occurs in higher organisms along with some lower followed by formation of fruits and seeds.
organisms.
24 Chapter 7 : How Do Organisms Reproduce

18. (i) Sperms are produced in seminiferous tubules present in The developing embryo will also generate waste substances
testes of male reproductive system which can be removed by transferring them into the mother’s
(ii) The long tail of sperms helps in quick movement or blood through the placenta. The development of the child
motility of sperms through the female reproductive tract. inside the mothers body takes approximately nine months
after that the child is born as a result of rhythmic contractions
(iii) Sperms are delivered from the site of their production by of the uterine muscles.
vas deferens, also known as sperm duct to the urethra in
24. Budding, fragmentation and regeneration are considered as
order to allow the passage of semen outside the body.
asexual types of reproduction because all of them involve only
19. (i) A-Fallopian tube, B-Ovary, C-Uterus, D-Cervix, one parent and gametes are not involved in the reproduction.
E-Vagina.
Regeneration in Planaria It can be cut into any number of
(ii) Fertilisation takes place in Fallopian tube. pieces and each piece grows into a complete organism. This is
20. known as regeneration. Regeneration is carried out by
specialised cells.
Uterus Fallopian tube
These cells proliferate and
produce large numbers
of cells. From this mass of
Ovary Ovary cells, different cells undergo
Endometrium changes to become various
Vagina cell types and tissues. These
Cervix changes take place in an
organised manner referred
(i) Ovum is produced by ovaries which are paired, oval-
to as development.
shaped organs.
25. (i) Asexual reproduction is a process which does not
(ii) Oviduct or Fallopian tubes are the site of fertilisation. involve fusion of two different gametes.
Oviduct have funnel-shaped opening near ovary and Three asexual modes of reproduction are:
carry ova or egg from ovary to uterus. (i) Fission The splitting of a unicellular organism into
(iii) Implantation refers to embedding of the embryo in the two or more separate, but identical daughter cells.
thick lining of uterus. (ii) Budding Division of parent organism by the
formation of an outgrowth called bud. The daughter
21. If the egg is not fertilised, it lives for about one day. Since, the cells may remain attached or can separate from the
ovary releases one egg every month, the uterus also prepares parent body.
itself every month to receive a fertilised egg. Its lining becomes (iii) Fragmentation The method in which multicellular
thick and spongy. organism breaks up into two or more smaller
This would be required for nourishing the embryo if fertilisation fragments.
had taken place. However, in absence of fertilisation, the (ii) Amoeba reproduces by binary fission
lining slowly breaks and comes out through the vagina as
blood and mucus. This cycle takes place roughly every month
and is known as menstruation. It usually lasts for about 2-8
days.
22. Placenta is a special tissue that helps the human embryo in
obtaining nutrition from mother’s blood. Parent Amoeba Elogation of Division of Two daughter
Structure Placenta is a disc-like structure embedded in the nucleus nucleus and cells
uterine wall. 
cytoplasm

(i) It contains villi on the side of the embryo. (iii) The main difference between fission and fragmentation
is that in fission, a unicellular organism breaks up to form
(i) It contains blood spaces, on mother’s side, which
two daughter organisms whereas in fragmentation, a
surround the villi.
multicellular organism breaks up into various fragments
Functions and each fragment develops into a complete organism
(i) It provides a large surface area for glucose and O2 to 26. (i) A-Bryophyllum-Vegetative propagation
pass from mother’s blood to the embryo. B-Plasmodium-Multiple fission
(ii) It also removes metabolic wastes from the embryo. (ii) Reproduction through spores gives several advantages
23. After implantation of the embryo it gets nutrition from to an organism. Some of them are as follows:
the mother’s blood with the help of a special tissue called Spores can remain dormant till favourable conditions
placenta. This is a disc, which is embedded in the uterine become available.
wall. It contains villi on the embryo’s side of the tissue. On the • Spores help an organism to tide over the bad phase.
mother’s side are blood spaces, which surround the villi. This • Spores can be spread through water, air or animals
provides a large surface area for glucose and oxygen to pass and thus is good for the spread of an organism to
from its mother to the embryo. more places.
BIOLOGY 25

(iii) Budding and regeneration. foetus take place. Rhythmic contractions of the muscles
Budding It is a process in which a bud develops as an in the uterus cause labour pain.
outgrowth due to repeated cell divisions at a specific site. This (ii) Structure of Placenta Placenta is a disc between
bud develops, gets matured and detaches from the parent cell uterine wall and embryo which is embedded in the
to become a new individual. uterine wall. It contains villi on the embryo’s side of
the tissue. On the mother’s side blood spaces are
Regeneration It takes place as specialised cells divide to
present, which surround the villi.
form large number of cells, undergo differentiation to become
various cell types and tissues.
Placenta
27.
Umbilical
(i) Pollen Tube Style cord
• A tube growing out • The middle elongat-
of pollen grain when ed part of the carpel, Embryo
Amnion
it reaches stigma. i.e. female part of a
• It transports male flower.
gametes from pol- • The attachment of
len grains to ovules. stigma to the ovary.
(ii) Binary Fission Multiple Fission
• The division of • The parental body Functions of Placenta It provides a large surface area
parental body divides into numer-
for glucose and oxygen to pass from the mother to the
into two identical ous daughter cells
embryo. It also removes the waste generated by embryo,
daughter cells at a simultaneously.
time. transferring it to mother’s blood.

(iii) Fragmentation Regeneration 29. (i) Placenta is extremely essential for foetal development
because it helps in nutrition, respiration, excretion, etc.,
• The method in • The growth of a
of the foetus through the maternal supply.
which multicellular whole new organism
organism breaks up from any of its body (ii) Blocking of vas deferens prevents passage of sperms,
into two or more part, i.e. single seg- hence, there is no fertilisation thus it prevents pregnancy.
smaller fragments. ment forming new
(iii) Wind acts as a pollinating agent because it helps in
individual.
transfer of pollen grains from anther to stigma of a flower.
(iv) Bud of Hydra Bud of Bryophyllum
(iv) Condoms prevent entry of sperms into vagina, hence
• It is seen during • This is present on
prevents pregnancy.
budding as an out- the leaf margins of
growth on the body Bryophyllum and (v) If Fallopian tube is blocked, sperm and egg do not meet
of Hydra which develop into a new and fuse and fertilisation does not take place.
gets fully grown plant when it comes
30. At the time of ovulation, a cycle of events takes place in the
and then detaches in contact with soil
uterus and vagina.
from the body and and other favourable
becomes a new conditions. (i) In case fertilisation occurs, the inner wall of uterus
individual. thickens to receive the developing zygote.
(ii) In case fertilisation does not occur, the thickened wall
(v) Vegetative Propaga- Spore Formation along with the blood vessels breaks down and moves out
tion of the vagina in the form of discharge called menstrual
• New plants are • Spores when fall on flow, which lasts for 2-8 days.
obtained from land, have the ability
different parts of to germinate and The cycle of events takes place in the ovaries and uterus
parent body like produce new fungal in every twenty eight days and marked by menstrual
leaves, stems, etc. colonies under fa- flow is called menstrual cycle.
vourable conditions.

28. Parts of female reproductive system Functions


(a) Ovaries Produce thousands of ova or egg cells. Secrete
female sex hormones like estrogen and progesterone.
(b) Oviduct (Fallopian tube) Carries ova or egg from ovary
to the uterus. It is the site of fertilisation.
(c) Uterus (Womb) Here, the growth and development of
26 Chapter 7 : How Do Organisms Reproduce

NTSE & OLYMPIAD QUESTIONS


1. (b) Saccharomyces cerevisiae commonly known as yeast, Therefore, it is only to be expected that the process

reproduces both asexually as well as sexually. Yeast of copying the DNA will have some variations each
reproduce asexually through a process known as time.
budding.
As a result, the DNA copies generated will be similar, but
2. (c) Female reproductive system consists of ovaries, fallopian may not be identical to the original DNA from parents.

tubes, uterus, vagina and external genitals. Ovaries are
11. (c) The fertilized egg or zygote gets implanted in the uterus
the paired structure and serve as primary female sex


after the process of fertilization. The uterus is a bag like
organs. Fallopian tubes are elongated tube like structures
structure that holds the embryo till the birth of the child.
that connect the superior part of uterus, fundus, with
ovaries. It is known as oviduct, these tubes propel the 12. (c) The sporangium of bread mould releases a large number


egg towards uterus and serve as site of fertilization. of spores that germinate on a suitable substratum to form
Uterus is thick walled muscular organ that projects into the hyphae.
vagina through its cervix. It serves as site of embryo Multiple hyphe branch and entangle to form a mat like
development. Vas deferens, also known as sperm duct, structure called the mycelium.
connects epididymis to urethra and serves to carry Fungi are saprophytes. Therefore, profuse growth is only
sperms from testis to urethra. possible in presence of a suitable substratum that supplies
3. (a) A chromosome is a single large macromolecule of DNA, large amount of moisture and nutrients, like bread.

and constitutes a physically organized form of DNA in a 13. (d) Flowers are a characteristic of flowering plants i.e.,


cell. It is a very long, continuous piece of DNA (a single Angiosperms. Other plants like Bryophytes, Pteridophytes
DNA molecule), which contains many genes, regulatory and Gymnosperms do not bear flowers. The function
elements and other intervening nucleotide sequences. of flower is sexual reproduction. For this purpose they
4. (d) Stamen are male reproductive organs. It contain anther have male and female gametes producing structures

which is site of pollen development. Inside anther male i.e., Anther and Ovary. A flower may be unisexual or
sporogenous cell differentiates and undergo meiosis to bisexual. For example, Cucurbits produce unisexual
produce microspores that develop into pollen grains. flowers while Solanaceae members like tomato, potato
produce bisexual flowers.
5. (b) Banana, yeast and Amoeba reproduce by the asexual

method. Banana reproduces by vegetative propagation, 14. (b) When Amoeba undergoes fission nucleus divides first

yeast reproduces by budding and Amoeba reproduces and then the cytoplasm.
by binary fission. 15. (b) Unisexual flowers are those, which bear only male or

6. (c) Anther is part of stamen (male sex organ) that produce female flowers, but never both together. They can either
possess stamen or pistil.

pollen (male gamete). Ovary is inferior part of pistil
(female sex organ) which contains ovule. Female As only one sex is present, it undergoes cross-fertilization,
gametophyte develops in ovule. where gametes are produced by two different plants.
7. (c) In biology, regeneration is an organism’s ability to Fruit develops from the female flower and hence, fruit

replace body parts. It is a specific method of healing that cannot be produced by the unisexual plants which
is ability to regrow lost limbs, severed nerve connections, possess male flowers (stamen).
and other wounds. It can be seen in the organisms of 16. (a) There is no bud on the parent cell.
Planaria and starfish. 17. (c) Regeneration is the development of a complete organism

8. (c) Pollination, fertilization, division of the zygote, the from the parts of the body of the organism which are

formation of embryo, seedling. accidentally cut during an injury. This process is found in
9. (b) When an individual organism increases in size via organisms like starfish, Hydra, Planaria, etc

cell multiplication and remains intact, the process is 18. (b) Binary fission is the form of asexual reproduction in

called “vegetative growth”. However, in vegetative singlecelled organisms by which one cell divides into two
reproduction, the new plants that result are new cells of the same size, used by most prokaryotes
individuals in almost every respect except genetically. 19. (c) Merogony is an asexual replication process used by

10. (c) The characters that are transmitted from parents to some Protozoan parasites that increases the number

offspring during reproduction show similarities with the of infective cells by multiple fission. It is the process by
parents because the offspring inherit them from their which the Plasmodium increases the infective cells.
parents. 20. (b) Vegetatively reproduced organisms show genetic

But, copying of DNA is a biochemical process, which is similarity in offsprings. This genetic similarity is because
not absolutely reliable. the offsprings are produced from a single parent.
BIOLOGY 27

21. (a) The Bryophyllum is a section in the plant genus 29. (c) Testes are male gonads or primary sex organs which



Kalanchoe of the Crassulaceae family. There are about produce sperms and secrete the male sex hormone
twenty to thirty species in the group, native originally of testosterone.
South Africa, Madagascar, Australia and Asia. The group
is notable for vegetatively growing small plantlets on 30. (d) Advantage of sexual reproduction is that it allows for


the fringes of the leaves; these eventually drop off and genetic variation. Stoppage of menstruation is termed as
develop into new plants. menopause. Animal development is not limited to the
period prior to birth hatching but it occurs throughout
22. (b) A sporangium (pl., sporangia) is a plant or fungal structure
different stages of their life.

producing and containing spores. Sporangia occur on
angiosperms, gymnosperms, ferns, fern allies, mosses, 31. (d) Merogony is an asexual replication process used by some


algae, and fungi. Microsporangia are the structures on protozoan parasites that increses the number of infective
the stamens of flowers called anthers, and the pollen- cells by multiple fission. It is the process by which the
producing structures on the microsporophylls of male
Plasmodium increases the infective cells. Buds produced
conifer cones. Megasporangia are the comparable
in the notches along the margin of Bryophyllum fall
“female” structures on these plants, associated with the
flower carpel and the megasporangial cone on the soil and develop into new plants. Hibiscus has
bisexual flowers.
23. (b) Variations are caused due to errors resulting the DNA

copying during the process of meiosis. Crossing over is 32. (d)
the process seen during meiosis resulting in the genetic 33. (c) Seed is formed from fertilised ovule having a dormant


variations. An error during crossing over results in an embryo and food reserve for subsequent development
error in the DNA copying. of embryo

24. (d) Androecium possess one or two whorls of stamens, each 34. (a) The embryo is having an embryonal axis or main axis

a filament topped by an anther where pollen is produced. called flagellum, one end of the flagellum possesses
Pollen contains the male gametes. Gynoecium possess radical or embryonic root while the other end has
one or more pistils. The female reproductive organ is the plummer or embryonic shoot.
carpel: this contains an ovary with ovules which contain 35. (a) Sponges are sessile because they are non-motile animals

female gametes. attached to some solid support.

25. (a) Placenta is a structure that is formed out of the fingerlike 36. (b) Sponges are porifers because their bodies have holes or

‘pores’ all over the body

projection both from the mother and child. It later on
becomes like a tube like structure that has both arteries 37. (a)
and veins. The embryo gets the nutrition from the 38. (d) A poikilotherm is an organism, whose internal

mothers blood with the help of the placenta. temperature varies considerably. It varies with the
26. (a) Testes is the male generative gland in animals. Male temperature of its surroundings. Mammals and birds
are homeotherms, therefore otter and whale being a

mammals have two testicles, which are often contained
mammal and penguin being a bird are homeotherms.
within an extension of the abdomen called the scrotum.
Tortoise being a reptile is a poikilotherm.
It produces the germ cells.
39. (a) Autotrophs are organism which make their food by

27. (c) A - (q); B - (s); C - (p); D - (r) themselves from simple substances.
28. (a) Vegetative propagation by buds or notches occurs in 40. (b) ‘Cross pollination’– the pollen grains are transferred from

potato. one flower to organisms made of more than one cell.
❑❑❑
8 ANSWERS & SOLUTIONS

MULTIPLE CHOICE QUESTIONS

1. (b) The sex gametes are always haploid. They have 23 Thus, out of 7524 F2 generation offsprings, the number
chromosomes. of round seeds will be calculated as follows: Out of 4
2. (a) A cross between a tall plant (TT) and short pea plant seeds, 3 seeds are round.
(tt) resulted in progeny that were all tall plants because Out of 7524, number of round seeds


tallness is the dominant trait. In F1 generation, the cross = (7524 X 3) / 4 = 5634.



between TT and tt will result in all tall plants. Thus,
12. (c) In pea plant, when pure tall pea plant (TT) is crossed
tallness is the dominant trait. Whenever two traits of a
with pure short (tt) plant, F1 generation will be all Tt (tall
character are crossed, the F 1 plants show only one of
plant). Thus, the ratio of pure tall plants (TT) to pure
the traits, this trait is called as the dominant trait. The
short plants (tt) is 1:1.
other trait (dwarfness) that does not express itself in F1
generation is called as the recessive trait 13. (c) In human males, all the chromosomes are paired
perfectly except one. These unpaired chromosomes are:
3. (d) The male sex chromosome is the deciding factor for the
Y-chromosome and X-chromosome.
gender of the child. If the X part fertilizes with the ovum,
girl is born. If Y fertilizes, then a boy is born. In human males, one pair called as the sex chromosomes

are unpaired. Here, one is a normal sized X chromosome
4. (b) A dominant inherent character can only be inherited by while other is a short Y chromosome. Women have a
the children. perfect pair of sex chromosomes, both called as X.
5. (d) A section of DNA that provides information for one 14. (c) Punnett square was used by GJ Mendel to determine the
protein is called the gene. law of inheritance in his experiments with pea plants.
6. (b) During fertilization, the mother always contributes an 15. (c) A trait in an organism is influenced by both maternal and
X chromosome while the father either contributes an X paternal DNA.
or a Y chromosome. Hence, the maleness of the child
16. (a) The number of sex chromosomes in the zygote of
is determined by the presence of Y chromosome in the
humans is one pair.
zygote contributed by the sperm.
17. (a) Evolution is a series of gradual changes in an organism,
7. (a) Charles Darwin is associated with theory of evolution.
wherein in the end, the organism is evolved and
8. (d) Characteristics can be called plants can photosynthesis, advanced.
the mango tree is multicellular, We have 2 eyes.
18. (c) Exchange of genetic material takes place in sexual
9. (a) All variations in a species do not have equal chances of reproduction.
survival. Some of the variations may be so drastic that the
19. (a) The change in the frequency of some genes which leads
new DNA copy cannot work with the cellular apparatus
to appearance of a new set of population without any
it inherits. Such a newborn cell dies soon. Depending survival disadvantage is called genetic drift.
on the nature of variations, different individual will have
20. (d) Methane, ammonia, hydrogen, water vapour
different kinds of advantage. Variations results from
change in genetic, composition. Advantageous variations 21. (d) Darwin’s theory of evolution can be described as:
are selected by environmental factors. This leads to within any population, there is natural variation. Some
evolution and speciation. In asexual reproduction, there individuals have more favourable variations than others.
is minimum variation that is due to small errors in DNA Even though all species produce a large number of
copy. offsprings, populations remain fairly constant naturally.
The struggle for survival within populations eliminates
10. (c) Palisade cells of leaves are somatic cells. Each gamete
the unfit individuals. This is called natural selection.
produced by the plant will have half the number
of chromosomes present in a palisade cell i.e. 14 22. (c) The process of selecting individuals with desired
chromosomes. characters by man is called Artificial selection.

11. (d) During mono-hybrid cross (cross with single 23. (b) The surprise products formed in experiment conducted
characteristics, the shape of seed), the ratio of round by Miller and Urey were amino acids.
seeds to wrinkled seed in F2 generation is 3:1, that is out 24. (c) Darwin’s theory does not include evolution Through
of 4 seeds, 3 seeds will be round and 1 will be wrinkled. Inheritance.
BIOLOGY 29

25. (d) In the context of evolution, the use of feathers by birds 43. (a) Because all the organisms have a common ancestor and
for flying means that birds have evolved from reptiles. they are closely related to the ancestors on the basis of
Dinosaurs had feathers but could not fly using them their morphological, analogical and genetic similarities.
. Brids, later adapted the feathers for filight. Since, Hence we can the more characteristics two species have
dinosaurs were reptiles, this means that birds have more closely they are related to each other and the
evolved from them. common ancestors.

1
26. (c) The theory of evolution is a shortened form of the term
“theory of evolution by natural selection,” which was MARK QUESTIONS
proposed by Charles Darwin and Alfred Russel Wallace
in the nineteenth century 1. The cross which occurs between the plants showing two
27. (b) The frequency of certain genes in a population changed alternate forms of a trait (character).
one generation. This is the essence of the idea of 2. Our modern understanding of how traits may be inherited
evolution. through generations comes from the principles proposed by
28. (b) The fossils are studied to determine the location and the Gregor Mendel in 1865.
origin of the organism. 3. Johannsen (1909) coined the term gene.
• The fossils of the ancient human species Homo
4. Gene which expresses itself is called dominant gene.


naledi were excavated in the southern regions of


Africa. 5. Genotype, the genetic constitution of an organism.
• With the help of research, the genetic footprint of

6. Sperm cells are haploid in nature. They contain 22
humans can be traced back to Africa. chromosomes which are autosomes.
29. (c) The process by which new species develop from the 7. The inheritance of character from parents to offsprings is
existing species is called speciation. called heredity.
30. (d) Classification involves hierarchical arrangement living 8. In one of the experiments with pea plants, Mendel observed
organisms into different categories on the basis of that when a homozygous tall pea plant (TT) is crossed with a
common inter-relationships between them. homozygous dwarf pea plant (tt), in the first generation, only
31. (b) Homologous organs represent common ancestry. It tall plants appear.
represents the evolution of closely related species from a 9. The chemicals which were essential for origin of life are
common ancestor. proteins and nucleic acid.
32. (c) They have same structures but different functions. 10. Turtle and Lizard are the organisms whose sex is depended
33. (a) Fossils are the remains of hard parts of life forms found upon the environmental factors
in rocks, e.g. tree trunks or skull. 11. Heredity units also known as genes which inherit character
34. (a) Wings of a pigeon and a bat are the example of analogous from parents to offsprings are called Mendalian factors.
organ 12. Russian biochemist A. I. Oparin has proposed that conditions
35. (c) Wings of a bat and a butterfly are considered as on the primeval earth billions of years ago were definitely
analogous organs. different from present conditions and the first form of life,
36. (b) Accumulation of variations due to mutations in the or self duplicating particles, did arise spontaneously from
genetic material over several generations forms new chemical inanimate or abiotic substances.
species. 13. Mendel carried out his key experiments using the garden pea,
37. (d) Animals with four limbs are called tetrapods, including Pisum sativum, as a model system.
amphibians, reptiles, birds and mammals 14. Homology indicates that there is common ancestry between
38. (d) Excavating, time-dating and studying fossils, as well a human hand and a wing of a bird. They have the same
as determining DNA sequences – have been used for fundamental structure but are different in external morphology
studying human evolution. and functions.

39. (d) The earliest member of human species, homo sapiens 15. F1 stands for Filial 1, the first filial generation seeds/plants or
can be traced from Africa. animal offspring resulting from a cross-mating of distinctly
different parental types.
40. (a) The preserved traces of living organisms are called
fossils. 16. Charles Darwin listed a number of putative human vestigial
features, which he termed rudimentary, in The Descent
41. (c) Those organs which have the same basic structure but of Man (1871). These included the muscles of the ear;
different functions are called homologous organs. wisdom teeth; the appendix; the tail bone; body hair; and the
42. (a) Those organs which have different basic structure but semilunar fold in the corner of the eye
have similar appearance and perform similar functions 17. Charles Darwin (1859) proposed the theory of natural
are called analogous organ. selection.
30 Chapter 8 : Heredity and Evolution

18. Round RR and yellow YY are dominant traits and green 3. Contrasting traits were used by Mendel and were classified as
yy and wrinkled rr are recessive traits so the offspring will be dominant or recessive
round Rr and yellow Yy dominating over wrinkle rr and green
Character Given Trait Contrasting Trait
yy traits
(i) Position of flower Terminal Axial

2 MARKS QUESTIONS (ii)


(iii)
Colour of flower
Shape of pod
White
Constricted
Violet
Full.

1. Fossils: They are remains or impressions of past organisms that 4. Let the dominant trait be represented by PP.
lie hurried in the rocks and other structures belonging to various Let the recessive trait be represented by pp.
ages.
Parents : PP × pp
Fossils Indicate Evolution
(1) Different types of organisms appeared in different ages.
F1 : Pp Pp Pp Pp
Many of them have later on disappeared. Some gave rise
All pink colour flowers, but hybrid.
to other organisms while a few are persisting even now.
(2) Fossils of different ages indicates the path of evolution,
2. Many of the variations have no immediate benefit to the species. Parents Pp × Pp
(self-fertilised)
They function as preadaptations which can be beneficial under
F2 : PP Pp pp pp
certain environmental conditions like heat tolerance variation, if
the temperature of the area rises. 3:1
3. Evolution indicates the forever evolving life on earth. Evolution 3 pink colour flowers : 1 white colour flower.
assures the continuity of various species & organisms on earth.
The accumulation of variations over a period of time is evolution.
Example: In a field there are red and green beetle. The crows are 5.
flying over the field found red beetles in the field to be eaten
which give green beetle an advantage over red beetles and more
the proportions of green beetles in the population would be in
the population will eventualy results in evolution of species to fit
the environment better.
4. Two causes are following:
(i) Genetic drift:- If the DNA changes are severe enough,
such as a change in the number of chromosomes,
eventually the germ cells of the two groups cannot fuse
with each other.
(ii) Natural selections:- If a new variation emerges in
which females are only able to mate with few males due
to their specific trait.

3 MARKS QUESTIONS
(i) In F2-generation, the combination of characters is Round
yellow/Round green=3
Wrinkled yellow=3, Wrinkled green=1
1. The two parents involved in sexual reproduction produce

gametes which fuse together forming a zygote. It gradually The ratio is 9 : 3 : 3 : 1


develops into a young child showing certain similarities with the (ii) In F1-generation, the production of all round yellow
parents. seeds explains that the round shape and yellow colour of
Since, a child inherits its characters from both the parents the the seeds were dominant traits over the wrinkled shape
resemblance with them is very close. and green colour of the seeds which segregated during
The grandparents and the child resemble less closely because a F2-generation.
gap of gene pool is created by the parents of the child.
Variations of two generations mixing together and addition of
new variations from parents, increases the difference between
them to a greater extent. Hence, a child resembles more closely
5 MARKS QUESTIONS

its parents than the grandparents. 1. Natural selection is preferential survival and differential
reproduction of individuals of a population which possess
2. (i) (a) Brown (b) Blue (c) Brown
variations that provide better adaptability to the existing
(ii) Eye colour in humans is an inherited trait. These are environment. Peppered Moth of England has two forms, light
traits that are present in the gene or DNA of an organism grey and dark grey. Prior to industrial revolution, tree trunks
and are passed on to their progeny. in the forests around
BIOLOGY 31

Manchester were light greyish green due to presence of lichens. 3. (a) Fossils: Fossils are remains, traces and impressions of
Most of the Peppered Moth found in the area were light the past organisms that are found in rocks and other
coloured with dark spots which could not be spotted easily by
structures of different ages.
predator birds. During 1845 to 1890, air pollution killed the
(b) Analogous: Though performing a similar function, the
lichen flora. The deposition of soot changed the colour of the
tree trunks to blackish one. Peppered Moth of the area also wings of the two are fundamentally different in their origin
exhibited switch over to melanic or blackish form. and structure. Wings of a butterfly are integumentary =
It provided better survival value against dark background. outgrowths while they are modified fore-limbs in bat.
The light grey form largely disappeared as it could be easily
4. (a) Appearance of trait of shortness in F2 generation
spotted by predator birds and devoured.
shows that the trait was present in F1 generation but
2. (a) Purple was not expressed while the trait of tallness expressed
(b) 25% itself. The trait which expresses itself in the presence of
(c) 1:2 its contrasting form is called dominant. The other trait
PP PP
purple × white which is unable to express its effect in the presence of its

contrasting trait is known as recessive.


Pp (Hybrid) (b) Genetic Drift: It is random change in gene frequency

in small isolated population owing to factors other than


PP Pp Pp PP
F2 Generation Purple natural selection that results in fixation of certain alleles
(Hybrid) (Hybrid) (White)
(pure)
and elimination of some others.

HOTS QUESTIONS

1. Sexual reproduction involves the fusion of male and female generation because it has been suppressed by the dominant
gametes, which leads to the mixing of characters of parents character which is the violet colour of flower.
and thus, causes variations in characters. 5. The trait which represents the tallness in a pea plant is

2. Variations occur in the genes of the organisms produced dominant over the another trait, shortness (dwarf), so progeny

due to the mutations, reshuffling of genes and inheritance of becomes tall.

acquired traits during the evolutionary process which make all 6. 23 pairs of chromosomes are present in human beings.
individuals different from one another. 7. Population is a group of one species that inhabits a particular
3. Environmental factors and mutations are the causes of area, individuals of which can interbreed. Evolution is
variation in asexually reproducing organisms. considered only if such a group of species or individuals shows

4. Heterozygous pea plants with violet flowers. a common advance trait not a single individual showing an

∙ In Mendelian experiment, violet flower (WW) is the dominant evolved trait.

character and white flower(ww) is the recessive character. 8. It is because of natural selection.

∙ When homozygous violet flowers are self breed with 9. Darwin hypothesised that evolution took place due to natural
homozygous recessive white flowers, the first filial generation selection.
produces 4 violet flowers with the genotype of heterozygous 10. Archaeopteryx indicates that birds have evolved from reptiles.
(Ww) and phenotype of violet coloured flower. They are the connecting link between birds and reptiles and
∙ The white flowers can not express its trait in the first have characteristics of both these organisms.
32 Chapter 8 : Heredity and Evolution

CASE BASED QUESTIONS

I. (1) (c) The allele for round seeds is dominated over that of
  (7) (a) A-D are round green and C-B are wrinkled yellow
 

wrinkled seeds and the plant with wrinkled seeds is type of seeds respectively.
homozygous. (8) (b) A and B are dominant traits.
 

(9) (b) A-B will be produced in maximum number in the F2


(2) (b) Rr × rr crosses will give round and wrinkled seeds in
 

generation.
 

same proportion.
(10) (d) Dihybrid cross
 

(3) (d) RR × rr cross can be used to determine the genotype


III. (11) (d) All of them have round seeds.
 

of a plant with dominant phenotype.


 

(12) (b) 1/4 of them have wrinkled seeds and 3/4 of them
(4) (a) The phenotypic ratio of the cross (F1) results in 3:1
 

have round seeds.




(round : wrinkled) i.e., 750, 250


(13) (b) If a genotype consists of different types of alleles, it is
(5) (b) The characters which appear in the first filial
 

called heterogyzous.
 

generation are called dominant characters.


II. (6) (a) A-B type of seeds are round in shape and yellow
 
(14) (d) The alternative form of gene is called allele.
 

in colour as round and yellow both constitute the (15) (c) 1:2:1 will be the genotypic ratio of the given F2
 

dominant character, hence expressed in generation. generation


A(round) and B (yellow) are dominant traits.

NCERT QUESTIONS

1. In a population of asexually reproducing species, the chances not passed to the DNA of germ cells. These are restricted to
of appearance of new traits due to variations is very low. And only somatic cells and are lost with the death of an individual.
the trait which is already present in the population is likely to Therefore, acquired traits like structural changes, experience,
be in higher percentage and would have been arisen earlier. intelligence cannot be transmitted to the progeny, i.e. these
Therefore the trait B, present in 60% of the population is the are not inherited.
trait which have arisen earlier. 6. Small numbers of surviving tigers are a cause of worry from
2. During reproduction, copying of DNA takes place, which the point of view of genetics because their decreasing number
is not 100% accurate, thereby causing variations. If these would cause a decrease in sett of genes, i.e. many genes will
variations are favourable they help the individuals to survive be eliminated thus, limiting the chances of variation during
and pass these variations to their progeny. Depending upon sexual reproduction and coming generations will not be able
the nature of variations, different individuals have different to see tigers at all.
advantages, which promotes their survival like bacteria which 7. Genetic variation, natural selection and reproductive isolation
can withstand heat will survive better in heat wave. could lead to rise of a new species.
3. The information is insufficient to tell whether the trait A or ‘O’ 8. No, geographical isolation will not be a major factor in the
is dominant. We can find out by assuming the following cases formation of new species of self-pollinated plants. This is
In case I: Let us assume that trait A is dominant. Father may because self-pollinated plants receive pollen grains from the
A O O O
have I or I and mother I I . In this case, 50% of the progeny same flower or another flower on the same plant and its
will have blood group A and 50% of the progenies will have distance from other plants hardly affects its reproduction.
A O
blood group ‘O’, when father’s blood group is I or I and Moreover, self-pollinated plants rarely show variations in
O O
mother is I I . characters.
In case II: Let us assume that ‘O’ is dominant. In this case, 9. Homologous characteristic can help to identify an evolutionary
we see that the child may have blood group ‘O’. Since, in both relationship between apparently different species. For
the assumptions the child can have blood group ‘O’, so we example, mammals have fourlimbs, as do birds, reptiles and
cannot infer which trait is dominant. amphibians. The basic structure of the limbs is similar though
4. There are different ways in which an individual with a it has been modified to perform different functions in various
particular trait may increase in a population. These include vertebrates. These are homologous organs.
genetic drift (i.e. the change in a type of genes in a population 10. Butterfly belongs to the phylum-Arthropoda, which have a
because of the random nature of reproduction) and natural chitinous exoskeleton. The bat belongs to class-Mammalia,
selection. which have wings made up of skin folds. Since, both butterfly
5. Acquired characters are structural, functional and behavioral and bat use their wings for flying, but do not share any
changes that an individual develops during its lifetime due to resemblance in their wings structure, they are not homologous,
a particular environment, disease, trauma, use and disuse, but analogous organs, i.e. having similar functions, but
conditioned and unconditioned learning, etc. These traits are dissimilar structure.
BIOLOGY 33

11. Fossils are the remains or impressions of organisms that lived Analogous organs have different basic structure and origin
in the ancient times. Fossils provide the evidence that the but have similar appearance and perform similar functions,
present animals have originated from previously existing ones e.g. wings of insects and wings of birds have different basic
through the process of continuous evolution. Fossils can be structure as the wings of insect is a fold of membrane and
used to reconstruct evolutionary history of an organism. wings of a bird are modified forelimbs, but have similar
Fossils are helpful in the study of evolution as: functions, i.e. flying.
(i) They give us an idea of the time in history when different 20. In order to find a dominant coat colour in dogs, homozygous
species were formed or became extinct. black (BB) male dog and a homozygous white (bb) female
(ii) Fossils also help us to trace the evolutionary history of dog are allowed to cross-breed in order to produce off springs
some animals. (F1-generation).
(iii) Fossils also indicate connecting links between two groups
If all the offsprings in the F1-generation are black, this concludes
of organisms, e.g. Archaeopteryx is a connecting link
that black colour coat is dominant over white colour coat in
between reptiles and birds.
dogs and if all the offsprings are white, the dominant colour
12. All human beings, even though they have different size, colour will be white.
and looks, belong to the same species (Homo sapiens) because
21. Life must have developed from the simple inorganic molecules
they have similar DNA sequences and have descended from
which were present on Earth soon after it was formed.
same ancestors.
Conditions on Earth could have given rise to more complex
Also, they are capable of reproducing among themselves. These organic molecules that were necessary for life. The first
variations may have arisen due to the environmental factors, primitive organism would have arisen from further chemical
mutation and mixing of characters during reproduction. synthesis. Miller and Urey’s experiment provided the evidence
13. Evolution is the generation of diversity due to environmental for the origin of life from inanimate matter.
selection. More and more complex body designs have emerged The organic molecules were assembled in an atmosphere
over time. Among bacteria, spiders, fishes and chimpanzees, similar to that thought to exist on early Earth over water.
we can say that the chimpanzees have a better body design This was maintained at a temperature just below 100°C and
because of their more complex body. sparks were passed through the mixture of gases to stimulate
Various organisms evolve in their own separate ways to give lightning.
rise to the current forms and have a basic difference in their At the end, complex carbon compounds were converted to
body design because of specialisation of all types of tissues. their simpler forms including amino acids, which makeup
The chimpanzees are best adapted to survive the present day protein molecules. This experimental setup demonstrates that
conditions and have proper division of labour in their body, life originated from inorganic molecules.
i.e. they have different organs for performing different vital 22. Sexual reproduction involves fusion of gametes. The offsprings
functions inside the body. show variations from their parents due to crossing over and
14. (c) The genetic makeup of the tall plant can be depicted as exchange of gene segments. They are not exact copies of
TtWW. their parents, due to recombination of parental genes. Also,
15. (d) Both organs in all options have same basic structural due to environmental factors certain favourable variations
design, but have different function appearance. are also produced. Due to production of variations, sexually
reproducing animals show very quick evolution. Whereas in
16. (a) Chinese school boy because both of us belong to the
asexual reproduction, organisms raised are the exact copies of
same species, i.e. Homo sapiens.
parents. They rarely show any variation.
17. From this study, we cannot make any inference whether light
23. The male and female reproductive cells divide by meiosis to
eye colour trait is recessive or dominant, because as both the
form haploid gametes. These gametes have equal genetic
parents have light colour eye, all the children will definitely
material. The zygote is formed by the fusion of male and
have light colour eye(through certain variations may occur).
female gamete, i.e. it has equal genetic contribution from male
18. The study of classification of various organisms gives us an idea and female parents as both parents contribute equal content
about the evolutionary history of the organisms. Organisms, of chromosomes. The individual is developed from the zygote
which have certain similar characteristics are placed in one thereafter.
group. It can be thus concluded that the organisms placed in
24. Yes, I agree. All the variations in a species do not have equal
one group may have evolved from common ancestors and chances of surviving in the environment. Depending on the
may have a common evolutionary history. nature of variations different individuals would have different
19. Homologous organs have the same basic structure and kinds of advantages. Selection of variants by environmental
origin but perform different functions, e.g. the forelimbs of a factors forms the basis of evolutionary process. The variations,
frog, a bird and a man have same basic design of bones, but which prove disadvantageous to an individual organism will
they perform different functions (frog uses them to jump, birds not survive because the environmental factors cannot support
use them to fly and man uses them to grasp things). this.
34 Chapter 8 : Heredity and Evolution

CBSE PAST YEAR QUESTIONS

1. Mendel selected true breeding tall (TT) and dwarf (tt) pea 8. Population is a group of one species that inhabits a particular
plants.When a tall pea plant is crossed with a short (dwarf) area, individuals of which can interbreed. Evolution is
pea plant, all the F1 hybrids are tall. (i.e., in this case, the gene considered only if such a group of species or individuals shows
causing tallness is dominant while the gene causing dwarfness a common advance trait not a single individual showing an
is recessive.).The trait expressing itself in the hybrid is the evolved trait.
dominant one. 9. It is because of natural selection.
(Mendel’s first law of inheritance states that when a pair of 10. The name of an organism is Planaria.
contrasting factors is brought in a hybrid, one factor inhibits
11. The two parents involved in sexual reproduction produce
the appearance of the other. The one which inhibits is the
gametes which fuse together forming a zygote. It gradually
dominant one and which is inhibited is recessive.)
develops into a young child showing certain similarities with
2. Mendel used a dihybrid cross between pure pea plants to the parents.
show that traits are inherited independently. • Since, a child inherits its characters from both the parents
He selected a pea plant with round yellow (RRYY) and the resemblance with them is very close.
wrinkled green (rryy) seeds. • The grandparents and the child resemble less closely
because a gap of gene pool is created by the parents of
In the F1 progeny it was found that all plants were round
the child.
yellow. But in F2 progeny some plants were round yellow
• Variations of two generations mixing together and
and some wrinkled green. However, there were plants which
addition of new variations from parents, increases the
showed new mixtures. Some of them were round with green
difference between them to a greater extent. Hence,
seeds, while others were wrinkled with yellow seeds. Thus, a child resembles more closely its parents than the
the round/wrinkled trait and green/yellow seed trait and grandparents
independently inherited.
12. Let the dominant trait be represented by PP.
3. A male germinal cell which forms gametes carries one X and
one Y chromosome while a female germinal cell carries only Let the recessive trait be represented by pp.
XX-chromosomes. Therefore, sex of the child depends upon Parents PP × pp
what happens during fertilisation.
F1-generation Pp Pp Pp Pp i.e, all pink colour flowers, but
(i) If a sperm carrying X-chromosome fertilises the egg, the hybrid.
child born will be female (XX). Parents self-fertilised Pp ×Pp
(ii) If a sperm carrying Y-chromosome fertilises the egg, the F2-generation PP Pp Pp pp
child born will be male (XY).
Ratio 3 pink colour flowers : 1 white colour flower
Thus, the sperm (the male gamete) determines the sex of the 13. (i) RR/rr
child.
(ii) Rr (hybrid)—Round
(iii) 3:1 (phenotypic ratio), 1 : 2 :1 (genotypic ratio)
XY XX
The name of this cross is monohybrid cross
14. Differences between acquired and inherited characters are as
X Y X
follows:

XX XY S. No Acquired Characters Inherited


Characters
4. No, because asexual reproduction involves only a single
(i) These characters develop These characters are
parent and does not require any other organism to carryout
in the organism during received by organisms
reproduction. So, the geographical isolation does not affect
their lifetime. from their parents.
its reproduction cycle. Moreover, asexually reproducing
organisms rarely show any variations in characters (ii) These do not bring about These bring about
any change in the genes change in genes of
5. Heterozygous pea plants with violet flowers. of organisms. organisms.
6. The trait which represents the tallness in a pea plant is (iii) These are lost with the These are transferred
dominant over the another trait, shortness (dwarf), so progeny death of the individual, to the next generation,
becomes tall. e.g. good physique of an e.g. fused and free
athlete, intelligence. earlobes.
7. 23 pairs of chromosomes are present in human beings.
BIOLOGY 35

15. (i) Yes, the next generation of the mouse will have tail 19. (i) When fossils of dinosaurs were discovered, it was seen
because removal of the tail cannot change the genetic that they had feathers on their body that provide them
make up of the mouse. insulation in cold weather, however they could not fly
(ii) (a) The presence of a long tail. using those feathers. Birds seem to have used feathers
(b) The presence of teeth. to fly later on. This, of course, means that birds are very
16. A trait in an organism, which has been acquired during the closely related to reptiles because dinosaurs were reptiles.
lifetime is known as an acquired trait, e.g. learning to swim,
Hence, it suggests that birds have evolved from reptiles.
driving, cooking, skating, reading, etc. The organisms are not
born with such traits, but cultivate them as they move through (ii) Insects, Octopus and Planaria are invertebrates, they
the life. These traits are not inheritable as they cannot be cannot be grouped together with vertebrates, as they
gained or passed on to the next generation. lack an internal skeleton with a backbone. Also, all of
The traits pass from parents to off springs through genes them belong to different phyla on the basis of different
present in gametes by process of reproduction. For a trait characteristics they possess. Just on the basis of one
to be inheritable it should be carried by genes (DNA)to the
characteristics, i.e. the presence of eyes, these organisms
off springs. Since, acquired traits cannot be passed to next
cannot be grouped together
generations, they do not result in evolution. e.g.
(i) A son of a weight lifter does not necessarily have a 20. The earliest members of human race are traced back to
muscular body as it is cultivated or have acquired trait. Africa. A couple of hundred thousand years ago, some of
(ii) Cutting the tails of mouse is not inheritable, hence, the our ancestors left Africa in search of food and habitat, while
future generations will have complete tails. others stayed back. After a few years, the migrants slowly
17. Speciation is the process by which new species develop spread across the world. They went forward and backwards
from the existing species. The four factors that could lead to with groups sometimes separating from each other, sometimes
speciation are: coming back to mix with each other during moving in and out
(i) Genetic drift (ii) Mutation of Africa thus causing the spread of our race in all parts of the
(ii) Natural selection (iv) Migration world.
Out of the following, geographical isolation will not be a major 21. Let us take the following example to justify the above
factor in the speciation of a self-pollinating plant species.
statement. Mendel crossed tall pea plants with dwarf pea
Because, new genes do not enter in the population and no
new species is formed.Hence, there is only a little variation plants.
possible. Speciation takes place only when variation is (Tall) (Dwarf)
combined with geographical isolation.
Parental Generation
TT
× tt
18. (i) If two groups of organisms of a single species never
exchange genes, i.e. do not undergo reproduction, there Gametes
T t
will be no evolutionary process. Reproduction forms the
basis ,of evolution, by fusing genes of two parents to F1 generation Tt
Selfing
form a new generation with new set of genes.
Tt × Tt
If there is no exchange of genes between members of the
species, new set of genes will not form, life will become Gametes T t T t
stagnant as old generations die and no new species is
present, thus making the organisms of that species
F2 generation
extinct. TT Tt Tt tt
(Tall) (Tall) (Tall) (Tall)
(ii) Diverse forms of life occur on Earth due to the evolutionary
Mendel’s observation: F1-generation contained all tall
process, the basis of which lies in reproduction. To study
this diversity classification of organisms is done. plants with genotype Tt, where T represents dominant trait
and t, a recessive trait.
Classification is the arrangement of organisms into various
groups and subgroups on the basis of certain similarities When F1-generation underwent selfing, the trait that was
and dissimilarities found in them. Based on these unexpressed in F1 (dwarf) was observed in some F2-progeny.
observations, organisms are placed in a hierarchy that Thus, both traits, tall and dwarf, were expressed in F1-
defines and brings out their relationships. The similarities
generation in the ratio of 3 : 1.
and differences are determined by characteristic features
present in organisms. 22. Evolution is the sequence of gradual changes from simple life

These include the ancestral and derived features (features forms to complex life forms, i.e. from primitive organisms that
developed due to the evolutionary changes, basic body lived over millions of years ago to new organisms that exist
design). today.
36 Chapter 8 : Heredity and Evolution

Evolution occurs by changes, improvement and modification fossils will be found in the upper layers or layers close to
of simple life forms. Fossils are the remains or impressions surface of the Earth.
of organisms that lived in the remote past. Fossils provide Depending on the depth of the fossils in the layers of
the evidence that the present animal have originated from Earth, the fossils are classified as recent or older.
previously existing ones through the process of continuous
25. In the exercise of tracing the family tree of species we consider
evolution. Fossils can be used to reconstruct evolutionary
certain things. Firstly, there are multiple branches possible at
history of an organism. The distribution pattern of fossils
each and every stage of this process. So, it is not as if one
shows that the ancient fossils present in the bottom rocks are
species is eliminated to give rise to a new one. Also, it is not
simple, while the most recent fossils found in the upper strata
as if the newly generated species are in any way better than
are more highly evolved. It means fossils form and become
the older ones. e.g. it is not true that human beings have
more and more complex as we proceed from earliest to recent
evolved from chimpanzees. Rather, both human beings and
rocks. It gives us an idea of time in history when different
chimpanzees had a common ancestor a long time ago.
species were formed or became extinct. Thus, fossils provide
us evidences in support of evolution. They probably evolved in their own separate ways to give rise
to the current forms.
23. Humans have, for more than two thousand years, cultivated
wild cabbage as a food plant and generated different vegetables Therefore, it is more appropriate to compare the process of
from it by selection. This method is artificial selection rather evolution with branches of a tree rather than with a ladder.
than natural selection. It has resulted in generation of the 26. Evolution does not occur because of a single change in
following vegetables due to some specific characteristics as: DNA or mutation. Organs are complex both structurally and
functionally; and are genetically regulated by many genes
(i) Cabbage selected for short distance between leaves.
working together. Evolution occurs in many stages by slow
(ii) Broccoli selected for arrested flower development and changes in DNA or mutation. The stages occurring during
thick stem. evolution can thus be identified by different structures formed
(iii) Cauliflower selected for sterile flowers forming a large from common ancestors. The original idea behind tracing
flower. such relationships was to identify the changes occurring in
DNA during reproduction.
(iv) Kohlrabi selected for a swollen edible stem.
(v) Kale selected for large leaves. The idea at present is called molecular, phylogeny. Comparing
the DNA of different species helps to give a direct account of
24. (i) Fossil A is a tree trunk changes that occurred in DNA during the formation of specific
Fossil B is a fish (Knightia) species. These changes occur during cell divisions and modify
the proteins that forms thereafter. These changes/variations
Fossil C is a dinosaur skull (Rajasaurus)
accumulate and pass on to the following next generation.
(ii) Fossil C, i.e. the dinosaur skull, was found only a few They can be used to trace back and identify the time of their
years ago in the Narmada Valley. initial occurrence. This approach is based on the idea that
(iii) The recent fossils refer to the present remains of the organisms distantly related will accumulate greater number of
organism who recently became extinct. The most recent differences in their DNA.

NTSE & OLYMPIAD QUESTIONS

1. (a) Variants by environmental factors like temperature form 4. (a) The Law of segregation states that when a pair of
the basis of evolutionary processes. contrasting factors or genes are brought together in a
heterozygote (hybrid), the two members of the allelic
2. (b) Reproductive processes generate individuals of similar
pair remain together without being contaminated and
design so, the rules of heredity determine the process by when the gametes are formed from the hybrid, the two
which traits and characteristics are reliably inherited. separate out from each other and only one enters each
gamete.
3. (a) Cloning is the process of producing similar populations
of genetically identical individuals from a single parent. Thus, Mendel’s law of segregation is based on the


separation of alleles during gamete formation


Clone is product of asexual reproduction.
5. (a) Classical plant breeding uses deliberate crossing of
Haploid is the term used when a cell has only one set of
closely or distantly related individuals to produce new


chromosomes. crop varieties or disease-resistant high yielding plants.


Autopolyploid is an organism having more than two

6. (d) Mendel proposed the laws of inheritance based on
sets of chromosomes, all of which were derived from experiments conducted on pea plants.
the same species. Genome is the genetic material of an 7. (c) Gene is a section of cellular DNA that provides
organism. information for one protein.
BIOLOGY 37

8. (c) There are two possible types of tall pea plants, i.e., 19. (c) Lamarck is best known for his ‘Theory of Inheritance of
homozygous and heterozygous. These have identical Acquired Characteristics’, first presented in 1801.
phenotypes and cannot be differentiated visually. If an organism changes during life in order to adapt


On selfing, however, the homozygous plants will produce



to its environment, those changes are passed on to its
only homozygous progeny but heterozygous plants will offspring.

produce phenotypic ratio of 3 : 1 between dominant and 20. (b) An experiment to prove that organic compounds were
recessive phenotypes. the basis of life, was performed by Miller.

9. (b) In animals sex determination is due to Y-chromosome. 21. (c) There are possibilities of errors in DNA copying due to
sexual reproduction causing variations.
10. (b) Darwin published his theory of evolution with compelling
22. (c) Evolution takes place because of both natural selection
evidence in his 1859 book On the Origin of Species.
and genetic drift.
11. (b) Genetic make up of the progeny is decided on the basis
23. (c) According to the theory of Neo-Darwinism, mutations
of the dominant and recessive traits of the parents. The together with natural selection are responsible for the
genetic makeup of the tall parent would have been origin of new species. Natural selection refers to the
TtWW. differential survival of organisms that is best suited to the
12. (a) A gene has two separate independent chromosome. environment due to the change in the genetic makeup of
Each cell will have two copies of each chromosome one organisms.
each from the male and female parents.. 24. (a) The giant nucleoprotein molecules were formed by the
union of nucleic acid and protein molecules. These
13. (c) In biology, evolution is the change in the characteristics
nucleoprotein particles were described as free-living
of a species over several generations and relies on the
genes. Nucleoproteins gave most probably the first sign
process of natural selection. Hence the evolution of of life.
the term in biology means that living things constantly
Hence Nucleoprotein gave most probably the first sign of
change.


life.
So, the correct answer is ‘Living things constantly
25. (d) Chemical evolution describes chemical changes on the
change’. primitive Earth that gave rise to the first forms of life.
14. (a) Homolgous structures have similar origin but disimilar 26. (d) The changes in non-reproductive tissues are the acquired
functions. traits and these cannot be passed on to the DNA of the
15. (b) In the 1800s, after Darwin first published his book, germ cells.
‘On the Origin of Species’, a British economist Herbert 27. (a) Speciation takes place on the basis of variation combined
Spencer used the term “survival of the fittest” in relation with geographical isolation..
to Darwin’s idea of natural selection as he compared 28. (a) If a round, green seeded pea plant (RRyy) is crossed
Darwin’s theory to an economic principle in one of his with wrinkled, yellow seeded pea plant, (rrYY) the seeds
books. This interpretation of natural selection caught produced in F1 generation are round and yellow.
on and Darwin himself even used the phrase in a later Parents: RRyy × rrYY
edition of ‘On the Origin of Species’. Gametes: Ry rY
F1 : RrYy (Round yellow)
16. (a) Colour variation can take place due to environment as
well. 29. (c) In human males, all the chromosomes are paired
perfectly except one. These unpaired chromosomes are:
17. (a) In human beings, males have XY chromosomes and
Y-chromosome and X-chromosome
females have XX chromosomes..
In human males, one pair called as the sex chromosomes
18. (d) Vestigial organs are the organs that once were useful in


are unpaired. Here, one is a normal sized X chromosome


an animal’s evolutionary past, but that now is useless or
while other is a short Y chromosome. Women have a
very close to useless. perfect pair of sex chromosomes, both called as X.
The human body has about 90 vestigial organs. These


30. (a) New species may be formed if DNA undergoes significant


included the muscles of the ear, wisdom teeth, the changes in germ cells and chromosome number changes
appendix, the tail bone, body hair, and the semilunar in the gamete.
fold in the corner of the eye (plica semilunaris). 31. (d) Characteristics are details of appearance or behaviour.
Malleus is one of the 3 bones present in the inner ear.

32. (d) Classification of species is done on the basis of cell design,
It receives vibrations from the tympanic membrane and specialisation of cell types and tissues and evolutionary
transmits this to the incus. relationship.
38 Chapter 8 : Heredity and Evolution

33. (b) 39. (a) Genetics is the branch of biology, that refers to the study
Parents
 : RRyy × rrYY of genes, genetic variations, and heredity in organisms.

(Round and green) (Wrinkled and Yellow) 40. (d) If one of the parents is homozygous AA, then it can form
only one type of gamete, having a dominant allele A and
Gametes: Ry × rY
other parent is also homozygous aa, so it can also form
F1 : RrYy only one type of gamete a. Fusion of these gametes will
F1 (Selfing) : RrYy
 × RrYy form only one type of individual Aa. Hence, probability
(Round and yellow) (Round and yellow) of having aa genotype is 0.

Gametes : RY, Ry, rY and ry ×


 RY, Ry, rY and ry 41. (d) The eye colour in Drosophila is sex linked and allele is
present on X chromosome. Thus a red eyed heterozygous
F2 : RRYY, RRYy, RrYY, RrYy RRYy, RRyy, RrYy,Rryy
female (XR Xr) will produce two types of gametes XR
(Round and yellow) (Round and yellow) and Xr while the white eyed male (Xr Y) will produce two
gametes ie Xr and Y. The R allele codes for red eye colour
RrYY, RrYy, RrYy, Rryy, (Round and yellow) and r allele codes for white eye colour. Y chromosome
rrYY, rrYy, rrYy (Wrinkled and yellow) does not has any allele for eye colour and R is dominant
over r.
rryy (Wrinkled and green)
Possible combinations of these four gamete types are
The new combination of characters are: Round yellow


XR Xr ie Red eye female,


and Wrinkled green. apart from parental combinations -
Round green and Wrinkled yellow. XR Y ie Red eye male,

34. (b) Gene is a particular sequence of DNA, that codes for Xr Xr ie white eye female,
a particular protein and is present on a particular Xr Y ie white eye male.
location on a given chromosome. Genes are located 42. (a) Complex organ may have evolved because it had
on chromosomes, which usually carry many genes. All survival advantage at intermediate stages.
the members of a species, have a same (not identical)
genes located on the same chromosomes. Thus, gene 43. (d) Because of natural selection and genetic drift, newly
sequence remains same on the chromosomes in different generated species are found to have more complex
individuals of a species. designs, though original species might not have vanished
completely
35. (c) Since, the parents are homozygous tall (TT) and dwarf
(tt), the progeny of F1 generation will be hybrids (Tt). In 44. (d) Theory of natural selection states that organic evolution
F2 generation, the tall and dwarf plants will be produced occurs through natural selection and accumulation
in 3 : 1 ratio. Out of the three tall plants, one tall plant of inheritable variations which provide structural and
will be homozygous tall (TT) and two tall plants will be functional superiority to some individuals over others in
heterozygous tall (Tt). The dwarf plant will be homozygous their survival and reproduction.
(tt), as dwarfness is a recessive character. Thus, the ratio Evolution prefers mutations, recombinations, gene


between homozygous tall and homozygous dwarf will be alterations, natural selection, crossing over, etc. Without
1 : 1. these key factors evolution will not occur.
36. (c) Similar homologous characteristics indicate common 45. (d) Three botanists - Hugo de Vries, Carl Correns and Erich
origins even in apparently different species whereas von Tschermak - independently rediscovered Mendel’s
similar analogous characteristics may not have common work in the same year, a generation after Mendel
origins.. published his papers. They helped expand awareness of
37. (d) Age of a fossil can be estimated by guessing how closer the Mendelian laws of inheritance in the scientific world.
it was to earth surface when found, by isotope dating or
46. (d) When a Rh-negative mother is impregnated by an Rh-
by comparing DNA of different species.
positive father and is carrying an Rh-positive baby, the
38. (d) Here, evolution means that birds have evolved from foetus Rh antigens will be perceived as foreign invaders
reptiles both mammals and birds evolved from reptiles, in mother’s body. It can initiate the protective mechanism
though not the same ones - mammals evolved from that can end up harming the child during second
the mammal-like reptiles, whilst birds evolved from pregnancy and is called as erythroblastosis foetalis. So,
dinosaurs. Some people maintain that birds actually Rh (+) man and Rh (-)woman should be avoided in
are dinosaurs, but this is not the case they have evolved biological marriages.
into different animals from their ancestors. Dinosaurs
47. (a) Paleontology is the study of fossil plants and animals.
were reptiles, belonging to the class Reptilia, whilst birds
It helps in establishing ancestral relationship between
belong to their own class, Aves. There is nothing odd or
surprising in them evolving into a different class of animal the organisms of the present to those of the past and to
to their ancestors certain fish evolved into amphibians, classify organisms accordingly.
certain amphibians evolved into reptiles, and certain 48. (a) Evolution can be described as generation and shaping of
reptiles evolved into birds and mammals. diversity by environmental selection.
BIOLOGY 39

49. (c) Study of evolution of human beings indicates that all 70. (a) Sex of the child is determined by inheritance of X or Y
humans originated from a single species that evolved chromosomes from males.
in Africa. Spreading across the world in stages with 71. (a) Mendel gave the law of independent assortment and
wide variations in environment and climate made the Charles Darwin gave the theory of Natural selection.
difference in colour, size and looks.
72. (a) The book ‘On the Origin of Species by means of Natural
50. (d) Basic structure of four limbs of birds, reptiles or Selection’ was published in the year 1859.
amphibians is similar to that of the mammals.
73. (a) Darwin formulated theory of natural selection in the year
51. (d) Structure and components of wings are different, but
1858.
look similar for their common function.
74. (a) Darwin’s theory is also known as theory of natural
52. (a) Extinct species must have existed at some stage describe
selection.
evolution.
75. (a) “Elephant is the slowest breeder becoming sexually
53. (c)
mature at the age of 30 and has a large gestation period.
54. (c) TtWW could be the genetic makeup of the tall parent. A single female reproduces on an average of 6 young
Since half of the progenies are short, the parent plant will ones during its lifetime.
also have a set of short genes; all progenies bore violet
76. (c) ‘Descent with modification’ is the central theme of
flowers, which means that violet colour is dominant over
evolution.
white.
77. (b) Every mutation or change in trait is caused by change in
55. (b) Bacteria have survived adverse habitats.
nucleotide sequence of DNA representing a gene.
56. (a) Becoming useful later for quite a different function than
78. (c) Adaptations can be either permanent (i.e. heritable from
abandoning is a welcome change.
generation to generation) or temporary (i.e., within an
57. (b) A - (p); B - (q); C - (r); D - (s) organism’s life span).
58. (a) A - (s); B - (r); C - (q); D - (p) 79. (a) Chromosomes are short, thick, condensed rod-like
59. (c) A - (q); B - (r); C - (p); D - (s) structures present in the nucleus which distribute genetic
60. (d) A - (q); B - (s); C - (p); D - (r) information to the daughter cells. They are capable
of self-replication before the cell division and thus are
61. (a) A - (r); B - (s); C - (p); D - (q)
passed to next generation. They contain genes on them
62. (b) A - (r); B - (s); C - (q); D - (p) which control the morphological and physiological
63. (c) A - (s); B - (p); C - (p); D - (r) properties of cells.
64. (c) Both (i) and (ii) statements are correct. 80. (c) Ear muscles of external ear in man are poorly developed.
(i) Life can originate on earth from pre-existing life

These muscles are useless which move external ear freely
only. and these muscles are called vestigial organs.
(ii) The atmosphere of the primitive earth was reducing.
81. (a) Life originates from pre-existing life forms and the


65. (d) Evolution is the unfolding of nature where in newer types process is termed as biogenesis. According to Oparin
of organisms develop from the pre-existing ones through and Haldane’s theory of biochemical origin of life, life
modification. Modifications occur due to accumulation originated spontaneously from non-living matter in
of variations. At the time of formation of earth lighter primitive atmosphere of earth (abiogensis). Assertion is
elements present on the surface were in their atomic correct.
state, viz, H, C, N and O.
The primitive reducing atmosphere of Earth was rich


66. (d) Variations develop during reproduction due to errors


in water vapour, carbon monoxide, carbon dioxide,
in DNA copying or mutations, chance separation of
nitrogen and hydrogen. UV light, high temperature
chromosomes during gametogenesis, crossing over and
and lightening served as sources of energy for chemical
chance pairing of chromosomes. and can be inherited.
reactions between above mentioned gases to form
Acquired traits are those variations which an individual
simple organic compounds which in turn produced
develops during its life time due to effect of environmental
more complex organic compounds and finally the first
factors, use and disuse of organs and conscious efforts.
living cell. The reducing atmosphere and absence of life
These traits ae non-inheritable..
on primitive Earth favoured origin of life which is not
67. (a) Exchange of genetic material takes place in sexual
possible in present day oxidizing atmosphere because
reproduction by involving male and female parents.
the compounds are readily oxidized as soon as they are
68. (c) Both (i) and (ii) formed. Thus, now life is not originating spontaneously
69. (b) (ii) only but only biogenesis exists. Reason is correct.
40 Chapter 8 : Heredity and Evolution

82. (a) Reproductive isolation is the absence of interbreeding 87. (c) Sex determination is a biological system that determines
between members of different species. This results in the development of sexual characters in an organism.
isolation of gene pool of section of the population from 88. (b) Evolution is a gradual development of more complex
rest inhibits gene flow ( a spread of genes between species from pre-existing simpler forms.
populations) and thereby leads to the formation of new
89. (a) Analogous organs have the property
species. Irrespective of rare frequency, all species exhibit
particular range of genetic recombination among the i. They have a similar function.
individual which can lead to merging of individuals of ii. They occur in unrelated organisms.
different species. Hence, reproductive isolation maintains iii. They show convergent evolution.
distinct identity of different lineages. Reproductive
90. (d) Dominant factor have the following property
isolation also prevents spread of morphological changes,
brought about by mutations, between different species. i. It is able to express its effect even in the presence of


Hence reproductive isolation is mandatory to maintain recessive factor.
distinct species and its absence can lead to merge of ii. It produces an effective or complete protein for


different species into one. expressing its effect.
83. (c) We have lost all the direct evidence of origin of life
because evolution does not proceed continuously in 91. (b) Natural selection have the following property
a particular direction. Continuation of old species also i. It is conducted by nature on a vast scale all over the


depends upon the environment. world.
ii. Traits selected for evolution are beneficial to the
84. (a) The banding pattern seen on stained chromosomes


species.
from humans and chimpanzee show striking similarities iii. Results are achieved over a long period of time.
which indicates that they have evolutionary relationships 92. (c) Peripatric speciation have the following property
(cytogenetic evidence). i. New species are formed in isolated small peripheral


85. (a) Allele is an alternative form of a gene that is located at a populations that are prevented from exchanging
specific position on a specific chromosome. genes with the main population.
ii. Genetic drift plays an important role here.
86. (a) Heredity is transmission of genetic characters from
parents to offspring or one generation to the next.
❑❑❑
13 ANSWERS & SOLUTIONS

MULTIPLE CHOICE QUESTIONS

1. (d) The flow of energy is not a functional component of an eco- 12. (c) An ecosystem includes both living and non living compo-
system. nents
2. (b) Crop field is an artificial ecosystem. It is an agricultural land 13. (a) Natural forest is the terrestrial ecosystem
created by man. 14. (d) Man is an omnivorous animal can eat both plants and ani-
Pond, lake and forests are natural ecosystem, as they are mals
self-sustainable and do not need human interference for 15. (a) Herbarium is an example of man-made ecosystem
their maintenance. 16. (c) Forest is the most stable ecosystem
3. (b) Organisms which synthesise carbohydrates from inorganic 17. (c) The ten percent law for the transfer of energy from one
compounds using radiant energy are called producers, e.g., trophic level to the next was introduced by Raymond Lin-
all green plants, blue-green algae. deman (1942)
4. (c) A food web is a network of inter-linked food chains operat- 18. (a) The build-up of toxic and chemical constituents like pes-

ing at various trophic levels. ticides, DDT, herbicides in the body of biotic organisms
5. (c) All green plants and few blue-green algae which can pro- through food web/chain is bio-magnification.

duce food by photosynthesis are called the producers. 19. (c) Too much exposure to the ultraviolet rays in the sun can

6. (b) Plants prepare food from sunlight and inorganic constitu- increase the risk of cataract and skin cancer.

ents like carbon dioxide and water. 20. (a) The full form of DDT is Dichlorodiphenyltrichloroethane. It

is known for its insecticidal properties.
7. (c) Herbivores occupy the second trophic level after autotrophs

21. (d) UV radiation is a highly damaging radiation, which when
or producers.

enters the environment can change atmospheric oxygen
8. (a) The study of flow of energy between various ecosystems
into ozone.

state that the autotrophs in a terrestrial ecosystem capture
22. (a) The use of chemicals like CFCs has endangered the ozone
about 1% of the energy of sunlight that falls on their leaves

layer.
and convert it into feed.
23. (d) Plastics bags and bottles come under the category of
9. (c) Producers convert sunlight into chemical energy and when

non-biodegradable products because they cannot be bro-

one form of energy is changed to another, some energy
ken down by natural process and tend to damage the safety
is lost to the environment in forms which cannot be used of the environment.
again.
24. (c) The increase in greenhouse gases in the environment leads

10. (d) The population of tiger decreases and the population of to global warming.
grass increases
25. (b) We should reduce the use of the plastic bags, bottles etc.
Explanation: If herbivores start missing, the producer pop-

because they are non-biodegradable.
ulation will increase and the population of carnivores will
decrease dramatically. 26. (d) All natural things produced by nature are biodegradable.
11. (d) In an ecosystem, along with the food chain, there is a loss of 27. (a) Depletion of ozone is mainly due to chlorofluorocarbons
energy and only 10 percent of the energy available at each (CFCs).
trophic level is passed on to the next trophic level. Thus, if
28. (d) Incinerators involve degradation of wastes by burning them
originally 5000 kJ of energy is available, at the next (or sec-
at high temperatures.
ond) trophic level 500 kJ of energy will be available, at the
next (or third) trophic level 50 kJ of energy will be available, 29. (d) The paper cups are preferred over plastic cups because be-
at the next (or fourth) trophic level 5 kJ of energy will be ing biodegradable they are not potential wastes.
available. 30. (b) Ozone is composed of three oxygen atoms.
42 Chapter 13 : Our Environment

1
16. They are steps or divisions of the food chain which are charac-
MARK QUESTIONS terised by particular methods of obtaining food, e.g., producers
(T1), herbivores (T2), primary carnivores (T3), etc
1. Jute bags or paper bags are prepared from biodegradable mate- 17. Phytoplankton → Zooplankton → Small fish → large fish
rials thus, lowering the environmental pollution. Plastic bags are 18. Global warming is a direct outcome of excessive burning of fossil
non-biodegradable and affect the environment adversely. fuels.
2. A food chain in an ecosystem is a series of organisms in which
each organism feeds on the one below it in the series.
Food chain 2 MARKS QUESTIONS

1. If we kill all the organisms in one trophic level, the lower trophic
level will grow more in number and the higher trophic level will
not survive. Hence, flow of energy from one trophic level to oth-
A fiid cgaub shows
the transfer of energy
er will not take place
within an escosystem 2. (i) Yes, the impact of removing all the organisms in a trophic


level will be different for different trophic levels. The lower


trophic level of an ecosystem has a greater number of indi-
viduals than the higher trophic levels.
3. Grass and plants belong to the 1st trophic level of a food chain. (ii) No, removal of all organisms of a trophic level will disturb
the ecosystem. Killing of higher trophic level organisms will
4. The energy flow through these organisms will follow a typical
cause explosion in the population of lower level organisms.
food chain pattern, i.e. producers → Herbivores → carnivores
3. Biological magnification refers to the increase in the concen-
→ decomposers.
tration of certain toxicants at each successive trophic level.
5. A food web consists of all the food chains in a single ecosystem. No, the levels of magnification will not be same in all trophic
6. Any undesirable change in the environment is considered as pol- levels.
lution. 4. The wastes like plastic burning discharge toxic components into
7. Ozone depletion will create holes in the stratosphere, which will the surroundings resulting in air pollution. The non-biodegrad-
allow the UV rays to enter the earth. The UV ray that reaches the able wastes are the prime reasons for land, air, soil, and water
skin can cause skin cancer. pollution.
5. If all the waste is biodegradable, then there will be no accumula-
8. Cow dung is a biodegradable substance and polythene is
tion of waste and the Earth would be a cleaner place to live. But
non-biodegradable substance
if, this biodegradable waste is too large in amount then its slow
9. Green plants are also called producers because only they can degradation.
synthesise organic food from inorganic raw materials with the

3
help of solar energy in the process of photosynthesis
MARKS QUESTIONS
10. (a) Biotic Components, e.g., producers, herbivores, carni-
vores, decomposers.
1. Advantage of cloth bags over plastic bags during shopping are:
(b) Abiotic Components e.g., climatic factors, edaphic fac-
(i) Cloth bags are biodegradable thus can be easily decom-
tors, topographic factors, inorganic nutrients and organic
posed by microorganisms.
substances.
(ii) Cloth bags can be used again, can be washed and do not
11. Chlorofluorocarbons (as refrigerants) and halons (in fire extin- cause any harm to the environment.
guishers). (iii) Plastic bags are non-biodegradable and hence can pollute
12. The ozone layer acts as a shield for life on Earth. Ozone is good the environment.
at trapping a type of radiation called ultraviolet radiation, or UV 2. Government of India is imposing a ban on the use of polythene
light, which can penetrate organisms’ protective layers, like skin, bags because they cannot be degraded naturally by the action
damaging DNA molecules in plants and animals.. of microorganisms. Because of their non-biodegradability, they
13. Biological magnification is an increase in the concentration of stay in the soil for a long time and continue to poison it with toxic
a chemical per unit weight of the organisms with the successive by-products that keep leaching from them.
rise in trophic level The advantages of using cloth and jute bags are as follows:
14. Bacteria and fungi obtain their nourishment from organic re- (i) They are environment-friendly as they are biodegradable.
mains and results in decomposition of organic matter. Therefore, (ii) They are renewable and can be easily recycled.
bacteria and fungi are called decomposers. (iii) They have more strength than polythene bags because they
are thick and can be used again and again
15. Ozone is a triatomic form of oxygen, O3. Ozone is formed in the
upper atmosphere by the action of ultraviolet (UV) radiations
over oxygen (O2)
BIOLOGY 43

3. A pond or lake is a natural ecosystem. They are self-sustaining It causes imbalance in the functioning of ecosystem and bio-
and complete. sphere. If organisms of one trophic level are eliminated, the
If any organism dies, there are microbes like bacteria and fungi organisms prior to that trophic level will flourish and increase
to decompose their bodies into simpler substances. An aquari- in number. Also, the organisms of the subsequent trophic
um, on the contrary is an artificial and incomplete ecosystem. level will sharply decrease, thereby creating an imbalance.
Abiotic components are not supplied naturally to it. It may not (ii) Vegetarian food chain is advantageous in terms of energy
have all the biotic components in it. If fish dies in an aquarium in
because it has less number of trophic levels.
the absence of decomposer, it will lie there as a rotten body pol-
luting the water of aquarium. Thus, an aquarium needs regular As we know, only 10% of the energy is transferred to the
cleaning next trophic level in a food chain, so if a person is vegetar-
4. In a freshwater aquatic ecosystem like a pond, the following or- ian then, he would have maximum amount of energy by
ganisms are included: consuming producers or plants in a food chain. Vegetarian
Producers: Rooted or large floating plants and the minute float- food chain gives ten times more energy than the non-vege-
ing plants, usually algae called Phytoplankton. tarian food chain.
Herbivores: The algae are eaten up by protozoans, (zooplank- 3. Some daily life eco-friendly activities are:
ton). (i) Save a tree, use less paper You can buy ‘tree-free’ 100%
Carnivores: The protozoans are eaten up by small fish. post-consumer recycled paper for everything from greeting
Large carnivores: Big fishes and fish-eating bird or animal eat cards to toilet paper.
small fishes. Paper with a high post-consumer waste content uses less
Food chain of a pond ecosystem pulp and keeps more waste paper out of landfills.
Algae → Protozoan → Small fish → Large fish/bird/animal (ii) Opt bamboo for hardwood floors Bamboo is consid-
(Producer) (Herbivore) (Carnivore) (Large carnivore) ered as an environmental-friendly flooring material due to
its high yield and the relatively fast rate at which it replen-
5. According to 10% law, only 10% of the energy entering a partic-
ishes itself. It takes just 4-6 years for bamboo to mature,
ular trophic level of organisms is available for transfer to the next
higher trophic level. compared to 50-100 years for typical hardwoods. Just be
sure to look for sources that use formaldehyde-free glues.
The flow of energy through a food chain is unidirectional and it
moves progressively through various trophic levels as follows: (iii) Reduce plastics, reduce global warming Each year,
Americans throw away some 100 billion polyethylene plas-
(i) Green plants capture 1% of energy of the sunlight that falls
tic bags being used as grocery and trash bags. Unfortunate-


on their leaves and convert it into food energy.


ly, plastics are made from petroleum, the processing and
(ii) When green plants are eaten by primary consumers, a great
burning of which is considered one of the main contributors


deal of energy is lost as heat to the environment. On an


to global warming, according to the EPA. In addition, send-
average only 10% of food eaten is turned into its own body
ing plastics to the landfill also increases greenhouse gases.
and made available for the next level of consumers.
Reduce, reuse and recycle your plastics are one of the best
(iii) Thus,10% can be taken as average value of the amount of
ways to combat global warming.


organic matter present at each step and reaches the next


(iv) Use healthier paints Conventional paints contain sol-
level of consumers.
vents, toxic metals and Volatile Organic Compounds

5
(VOCs) that can cause smog, ozone pollution and indoor
MARKS QUESTIONS air quality problems with negative health effects, according
to the EPA. These unhealthy ingredients are released into
1. Organisms which breakdown the complex organic compounds the air, while you are painting, drying of paint and even
present in dead and decaying matter are called decomposers, after the paints are completely dry. Opt for zero or low VOC
e.g. certain bacteria and fungi. paint, made by major paint manufacturers today.
Decomposers act as cleaning agents of environment by decom- (v) Use compost Instead of using synthetic fertilisers, com-
posing dead bodies of plants and animals. They also help in post provides a full complement of soil organisms and the
recycling of materials, replenishment of soil’s nutrients, etc. balance of nutrients needed to maintain the soil’s health.
Healthy soil minimises the population of weeds. It is a key
The consequence of their absence in an ecosystem can be disas- to produce good quality plants, which in turn can prevent
trous. The dead bodies would persist for long, leading to their many pest problems from developing.
accumulation and thus, polluting the environment. 4. (i) The producers convert solar energy into chemical energy


Nutrients associated with these remains will not be returned back in the form of organic compounds. The primary consumers
to the environment. As a result, all the nutrients present in soil, (herbivores) derive their nutrition from the producers.
air and water would soon be exhausted and the whole life cycle So, the energy that is captured by the producers does not
of organisms will be disrupted. revert back to the Sun and the energy transferred to the her-
2. (i) Undesirable activities of man eliminate growth of organisms bivores does not come back to the producers. It just keeps
on moving to the next trophic level in one direction. That is
belonging to one or more trophic levels in a food chain.
why the flow of energy in the food chain is always unidirec-
Thus, the food chain gets shortened, e.g. hunting tigers for tional.
their skin, etc.
44 Chapter 13 : Our Environment

(ii) A large number of pesticides and chemicals are used to pro- (i) Soil degradation: Extensive cropping causes loss of soil
tect crops from pests and diseases. Some of these chemicals fertility. Also, over the time it can lead to soil erosion and
are washed down from the soil, while some enter the wa- finally to desertification
terbodies. From the soil, they are absorbed by plants along
(ii) Pollution: Use of synthetic chemical fertilisers and pesti-
with water and minerals, and from the waterbodies, they
cides leads to soil, water and air pollution.
are taken up by aquatic plants and animals. This is how
these chemicals enter the food chain. (iii) Water shortage: Excess use of ground water for agricul-
As human being occupy the top level in any food chain, ture lowers the water level. This results in acute water short-
these chemicals get accumulated in our bodies in maximum age at many places.
amount (iv) Biomagnification: The chemical pesticides, being
5. Some harmful effects of agricultural practices on the environ- non-biodegradable accumulate in organisms in increasing
ment are as follows: amounts at each trophic level.
(v) Deforestation: Indiscriminate cutting of trees for agricul-
ture has resulted in loss of habitat for wildlife. Thus, it also
causes damage to natural ecosystem.

HOTS QUESTIONS

1. Ozone occurs in two layers of the atmosphere. The layer closest 7. The role of consumers is to obtain energy by feeding on other
to the Earth’s surface in the troposphere. Here, ground- level or organisms and sometimes transfer energy to other consumers in
“bad” ozone is an air pollutant that is harmful to breathe and it a ecosystem.
damages crops, trees and other vegetation. 8. Advantage of Ozone–When it is in the stratosphere it does not
2. A natural ecosystem has a diverse amount of species and plants, allow the ultraviolet radiations to reach the earth as UV radia-
whereas artificial ecosystems are limited. Natural ecosystems tions cause skin cancer and cataract.
are self-sustaining and result from spontaneous natural reaction, Disadvantage of ozone: On ground level ozone is act as
while artificial ecosystems require the assistance of humans. poisonous gas.
3. Plastic bag is not acted upon by decomposers as it cannot be 9. An ecosystem is a group or community composed of living and
broken down into simple components so it is called non-biode- non-living things and their interactions with each other. They can
gradable while paper gets decomposed. be natural as well as artificial. Every ecosystem has two compo-
4. Two decomposers are bacteria and fungi. Two producers are nents, namely, biotic components and abiotic components
grass and leaf. 10.
5. Pesticides kill insects and pests thereby protect the crops but
these pesticides remain on the crops which enter the food chain
Biodegradable Non-biodegradable
and get accumulated in the organisms at the top most tropic
level that causes diseases. (i) Biodegradable wastes Non-biodegradable
are those substances wastes are those
6. Two artificial ecosystems are the crop field ecosystem and the that degrade or break substances that do not
aquarium ecosystem. down naturally. degrade easily.
(ii) e.g., plants, animals, e.g., plastic

CASE BASED QUESTIONS

I. (1) (b) Green plants in a terrestrial ecosystem con-


 (2) (c) Movement of energy is unidirectional and matter


vert 1% of the total solar energy falling on it is repeatedly circulating.


into food. These are the producers or autotro- (3) (c) X eating curd/yoghurt should be considered as


phs. Then there are Primary consumers, Sec- occupying third trophic level. Producers or green
ondary consumers and Tertiary consumers in plants (first trophic level) are consumed by herbi-
the food chain. In each of these step only 10% vores (second trophic level) and from them curd/
energy is transferred but from sun to the plants yoghurt (made from dairy breed) is consumed by
only 1% of the energy is converted into food third trophic level organisms like man.
Thus, if 10,000 J of solar energy falls on
II. (4) (a) A network of interconnected chain is called food-
green plants in a terrestrial ecosystem,


web
the energy converted into food energy
= 1% of 10000 J = 10000/100 J = 100 J
BIOLOGY 45

(5) (c) A food chain is a linear network of links in a food


 (10) (d) Enhanced UV radiations would affect humans


web starting from producer organisms and ending and other animals causing, skin cancer, blindness
at an apex predator species, detritivores, or de- and increased chances of cataracts in the eyes and
composer species malfunctioning of the immune system.
(6) (d) Grass is the producer as it is a plant that can pre-


(11) (d) Ozone-depleting substances (ODS) are substanc-




pare its own food. es that react with ozone present in the stratosphere
(7) (d) All the statements above are correct regarding

and destroy the same. The major Ozone-depleting
green house effect. substances are chlorofluorocarbons (14% of total
(8) (c) Accumulation of carbon dioxide and depletion of

depletion), nitrogen oxides (3.5% depletion), sul-
ozone lead to the causing of green house effect. phur dioxide, halon, carbon tetrachloride, methyl
III. (9) (c) Ozone layer is present in Stratosphere layer of the

chloroform, chlorine, etc
atmosphere.

NCERT QUESTIONS

1. Certain substances are easily acted upon by the enzymes of sap- 6. Ozone is a triatomic molecule i.e. made up of the atoms of oxy-
rophytes and get converted into simpler substances, hence are gen joined together. Its molecular formula is O3.
biodegradable. Whereas certain substances like plastic, etc., can- It can affect any ecosystem in the following ways:
not be degraded by the action of enzymes and thus, are called (i) It protects against ultraviolet rays if, present in strato-
non-biodegradable. sphere.
2. The two ways in which biodegradable substances would affect (ii) Ozone dissipates the energy of UV rays by undergoing
the environment are: dissociation followed by reassociation.
(i) Their degradation may release certain gases in the 2O3 → 3O2 + Energy
atmosphere thereby, polluting the environment. (iii) In atmosphere, it is highly toxic and cause injury to mu-
cous membranes, eye irritation and internal haemor-
(ii) They may become breeding places of flies and many
rhages in animals and humans.
other pests, thus causing diseases.
7. We can reduce the problem of waste disposal by these methods-
3. The two ways in which non-biodegradable substances would af-
•  By minimizing the use of disposable items and promoting
fect the environment are: the use of recycled articles.
(i) They make the environment poisonous and unfit for sur- •  Separating biodegradable and non-biodegradable waste
vival of living forms of life. before dumping them.
(ii) They block the transfer of energy and minerals in the • Recycling the non-biodegradable waste material.
ecosystem. 8. (i) Yes, the impact of removing all the organisms in a trophic


level will be different for different trophic levels. The lower


4. The transfer of food or energy takes place through various levels
trophic level of an ecosystem has a greater number of indi-
in the food chain, which are known as trophic levels, e.g.
viduals than the higher trophic levels. Removal of produc-
   Trees   →   Rabbit    → Snake
     →    Hawk ers will affect all the organisms of successive trophic levels
   (First      (Second     (Third      (Fourth and it will threat their survival. The removal of higher tro-
trophic level) trophic level) trophic level) trophic level) phic level will lead to increase in organisms of lower trophic
level and the organisms of higher trophic level will die due
[Producers] [I consumer] [ll consumer] [III consumer]
to the shortage of food.
5. Organisms that feed on dead plants and animals are called (ii) No, removal of all organisms of a trophic level will disturb
decomposers, e.g. bacteria, fungi, etc. They breakdown the the ecosystem. Killing of higher trophic level organisms will
complex organic compounds present in the dead remains into cause explosion in the population of lower level organisms.
This will adversely affect the ecosystem.
simpler substances and obtain nutrition from them. These sub-
stances are released into the soil and the atmosphere. 9. Thinning of ozone layer present in stratosphere is called deple-
tion of ozone layer. Due to depletion of ozone layer, harmful
Thus, they play the following roles: ultraviolet radiations can reach the surface of Earth, which may
(i) They help in recycling of materials, replenishment of the lead to skin diseases, cancer, etc. To reduce the depletion of
soil’s nutrients, etc. ozone layer, use of chlorofluorocarbons has been minimised. In
(ii) They clean our surroundings by decomposing dead or- 1987, the UNEP has passed an agreement to freeze CFC pro-
ganisms and organic wastes. duction at 1986 levels.
46 Chapter 13 : Our Environment

CBSE PAST YEAR QUESTIONS

1. Biodegradable materials are broken down by micro-organisms 6. A large number of pesticides and chemicals are used to protect
in nature into simple harmless substances. Non-biodegradable our crops from pests and diseases. Some of these chemicals are
materials need a different treatment like heat and temperature washed down into the soil, while some enter in the water bodies.
for disposal and hence, both should be discarded in two different From the soil, they are absorbed by plants along with water and
dustbins. minerals and from water bodies, they are taken up by aquatic
plants and animals.
2. Affluent people use resources recklessly, which leads to their de-
This is how these chemicals enter the food chain. Since, these
pletion and generation of excess waste. Due to their high living chemicals cannot be decomposed, they accumulate progressive-
standards, such affluence has terrible consequences on the envi- ly at each trophic level. As the food chain proceeds, the concen-
ronment. tration of pesticides also increases. This increase in the concen-
3. Government of India is imposing a ban on the use of polythene tration of harmful chemicals at each step of the food chain is
bags because they cannot be degraded naturally by the action called biomagnification. That is why food grains, such as wheat
of micro-organisms. Because of their non-biodegradability, they and rice, vegetables, fruits and even meat are found to contain
stay in the soil for a long time and continue to poison it with toxic pesticide residues.
byproducts that keep leaching from them. 7. The household waste produced from various activities is called
garbage and its proper disposal is done in such a way that it does
Also, they do not allow water to seep in, as they are waterproof.
not affect the environment. This is called garbage management.
These polythene bags, when accidentally eaten by stray animals,
Methods of waste disposal include:
can harm them and can even lead to their death. Jute and cloth
(i) Recycling The processing of certain wastes to form new
bags can be used in place of polythene bags. The advantages of
products is called recycling, e.g. paper, glass, polythene,
using cloth and jute bags are as follows:
etc., are recyclable. They can be used to make cards, dec-
(i) They are environment-friendly as they are biodegrad- orative items, etc.
able. (ii) Composting It is the process of collecting biodegrad-
(ii) They are renewable and can be easily recycled. able wastes like leftovers of food items, peels, etc., and
burying them in a pit. The product is used as manure.
(iii) They have more strength than polythene bags because
(iii) Incineration It is burning of a substance at high tem-
they are thick and can be used again and again.
perature to form ash.
Thus, use of jute and cloth bags can help to reduce pol- (iv) Landfills Dumping of waste in low lying areas is called
lution. landfill.
4. According to 10% law, only 10% of the energy entering a partic- (v) Sewage treatment In sewage treatment plants, the
ular trophic level of organisms is available for transfer to the next sewage is processed and decomposed.
higher trophic level. 8. (i) The producers convert solar energy into chemical energy


The flow of energy through a food chain is unidirectional and it in the form of organic compounds. The primary consum-
ers (herbivores) derive their nutrition from the producers.
moves progressively through various trophic levels as follows:
According to the energy transfer law, only 10% of energy is
(i) Green plants capture 1% of energy of the sunlight that transferred from one trophic level to the other.
falls on their leaves and convert it into food energy. So, the energy that is captured by the producers does not
(ii) When green plants are eaten by primary consumers, a revert back to the Sun and the energy transferred to the her-
great deal of energy is lost as heat to the environment. bivores does not come back to the producers. It just keeps
On an average only 10% of food eaten is turned into its on moving to the next trophic level in one direction. That is
own body and made available for the next level of con- why the flow of energy in the food chain is always unidirec-
sumers. tional.

(iii) Thus,10% can be taken as average value of the amount (ii) A large number of pesticides and chemicals are used to
of organic matter present at each step and reaches the protect our crops from pests and diseases. Some of these
chemicals are washed down from the soil, while some enter
next level of consumers.
the water bodies. From the soil, they are absorbed by plants
5. (i) Decomposers are essential in a biosphere as they break-


along with water and minerals, and from the water bodies,
down dead complex organic matter into recyclable simpler they are taken up by aquatic plants and animals. This is
compounds. Also, they help in returning the nutrients to the how these chemicals enter the food chain.
nutrient pool.
As these chemical not decompose, they accumulate pro-
(ii) The flow of energy through different steps in the food gressively at each trophic level. This increase in the con-
chain is unidirectional. The energy captured by autotrophs centration of harmful chemicals with each step of the food
(plants) does not revert back to the solar input and it passes chain is called biomagnification. As human being occupy
to the herbivores and moves progressively through various the top level in any food chain, these chemicals get accu-
trophic levels. mulated in our bodies in maximum amount.
BIOLOGY 47

NTSE & OLYMPIAD QUESTIONS

1. (b) Secondary consumers are carnivores which feed on pri- products of organisms into simpler inorganic substances,
mary consumers and tertiary consumers are top carnivores which get mixed with soil as minerals.
that feed on other carnivores. 14. (a) It follows that the carnivores (secondary consumers) that
2. (b) A food chain is series of plants/animals which are interre-
 feed on herbivores and detritivores and those that eat other
lated in the form of organism being eaten as food by the carnivores (tertiary consumers) have the lowest amount of
other. A food chain consists of producers, consumers and energy available to them
decomposers. It begins with the producers. These include 15. (d) Disposable plastic plates should not be used because its
green plants which produce their own food. The animals waste is non-biodegradable and cannot be broken down
eating plants are called as primary consumers. They are into its base compounds by micro-organisms, air, moisture
herbivores. Animals eating primary consumers are called as or soil in a reasonable amount of time. Non-biodegradable
secondary consumers. They are carnivores. waste is an environmental concern, as it threatens to over-
3. (a) The substances which get decomposed by the action of
 whelm landfills and create disposal problems.
bacteria or some other saprophyte or some physical pro- 16. (c) All ecosystems are characterised by a unidirectional flow of
cesses under atmospheric conditions are biodegradable energy. At each trophic level, most of the energy available is
substances. utilised for respiration and excretion and only ten percent of
4. (a) Polythene is a non-biodegradable substance as it not affect- the available energy is passed on to the next level. Because
ed by bacteria. only ten percent of the available energy can be passed on
5. (b) All the living and non-living organisms interact with each to the next trophic level, higher trophic levels have substan-
other in an ecosystem. tially less energy content and the number of trophic levels in
a food chain is limited.
6. (c) Biotic component comprising living organisms and Abi-
otic component comprising physical factors constitute an The lower the trophic level higher will be energy content.
eco-system. Hence, the greatest amount of energy is expected in trophic
level T1.
7.  (a) Abiotic components consist of the non-living components
like light, temperature, water, oxygen,carbon, nitrogen and 17. (b) Decomposers are micro consumers and include certain
minerals. Various important abiotic factors have been clas- bacteria and fungi. They are known as saprophytes and re-
sified as follows: lease the minerals trapped in organic remains of plants and
animals. The process of decomposition occurs outside the
Climatic factors: These include light, temperature, precip-
body of decomposers.
itation, atmospheric humidity and wind.
They release enzymes on the organic dead remains which
8. (a) Arthur George Tansley coined the term ecosystem’ in 1935
decompose them into simple inorganic substances which

to describe the interactions between organisms and with


are absorbed by decomposers.
their environment. So, the correct answer is ‘Tansley’.
18. (b) All green plants and certain blue green algae do photosyn-
9. (b) Chlorofluorocarbons:-
thesis in presence of sunlight and chlorophyll.
• Chlorofluorocarbons has been replaced recently to
HCFC’s. 19. (c) Food chains represent the interdependence of organisms
• The main advantage that HCFCs and HFCs have over for their fulfilling their energy needs.
CFCs is that they are much less stable 20. (a) Each step or level of the food chain forms a trophic level.
• and more reactive with their additional hydrogen 21. (c) Various components of the environment involves the flow
atom(s), of energy.
• meaning they can usually break down in the tropo-
22. (c) The diagram is a food pyramid and starts with producers or
sphere before reaching the stratosphere and attacking
plants at the base and ends with the consumers. Here, the
the ozone.
plants that primarily synthesize energy are consumed by the
10. (c) Continuous interaction between living and non living things cricket, which is eaten by the frog. The frog is eaten by a
affects the growth, reproduction and other activities of the snake at the apex. Hence the snake has the least biomass.
living organisms.
23. (c)
11. (d) Both natural ecosystems like forests, ponds and lakes or ar-
24. (b) Green plants synthesise their own food from carbon dioxide
tificial ecosystems like gardens and crop fields are habitat to
and water in the presence of sunlight.
several living organisms.
25. (c) The UV-rays have extremely harmful effects on human
12. (a) Herbivores eat plants and get eaten by carnivores. Parasites
beings, other animals as well as plants. It can cause skin
feed on plants, herbivores and carnivores.
cancer, damage the eyes by causing cataract, also damage
13. (b) Decomposers are the micro organisms that break-down the mmune system by lowering the body’s resistance to
complex organic substances of dead remains and waste diseases.
48 Chapter 13 : Our Environment

26. (b) There are four levels: Level 1 of producers, Level 2 of her- 35. (a) ‘Ozone is a poisonous gas’ is a property and not its forma-
bivores, Level 3 of carnivores and Level 4 of vultures. The tion.
levels are not clearly demarcated as interlinking also takes 36. (c) Ozone layer helps in stopping harmful UV radiations from
place. the Sun to enter the earth’s atmosphere. Excess amount of
27. (a) Green plants capture 1% of the energy of sunlight that falls chlorofluorocarbons (usually from refrigeration appliances)
on their leaves for carrying out photosynthesis. Rest of the in them atmosphere is found to endanger the ozone layer .
energy is used to warm or evaporate water, carrying out 37. (d)
various other natural phenomenons or even get reflected by
38. (a) Global warming
shining snow.
39. (c) CO2
28. (b) The energy transferred from a lower level to the higher level
40. (b)
only and not vice versa
41. (c) Pyramid of biomass in a pond ecosystem An inverted pyra-
29. (a) Wastes which are broken down and disposed off naturally
mid is formed when the number of individuals or biomass is
by saprophytes or decomposers are called biodegradable
minimum and increases at each trophic level. The pyramid
wastes. They are generally organic wastes, e.g., sewage,
of biomass in a pond ecosystem is an example of an invert-
livestock waste, used tea leaves, waste paper, leather, wood
ed pyramid because the biomass of phytoplankton may be
and jute articles, food leftover, etc.
smaller than that of zooplankton
30. (a) Decomposers play the main role in recycling the elements
42. (b) In an ecosystem plants produces their own food by utilising
in the environment. They release enzymes to break down
solar energy, water and carbon dioxide, which is then trans-
complex substances into simple substances. The energy re-
fer to the successive trophic levels. Some of the minerals
leased in this process is utilized by decomposers.
are locked in living systems as constituents of protoplasm.
The things which can be degraded by decomposers are These minerals are returned to environment through the
termed as ‘bio-degradable’. But some human-made things decomposition of death and decay of organisms.
cannot be degraded by decomposers, as they do not pos-
43. (d) Decomposition is the chemical breakdown of complex or-
sess enzymes to degrade such things.
ganic remains by decomposers brought about by bacteria
These things are termed as ‘non-biodegradable’. These and fungi.
things are plastic, paint, aluminium container and lead bat-
44. (b) Disposable packaging materials are usually made from
teries etc.
non-biodegradable substances.
31. (d) Non-biodegradable items are those which cannot be acted
45. (a) Disposable paper cups are biodegradable. Clay gets mixed
upon by detrivores. Thus, such items cannot be degraded
with soil only after a long time.
or decomposed. In a way, they are a permanent addition to
the environment and cannot be done away with. For exam- 46. (d) Food chain can have four levels for energy transfer of which
ple, polythene, thermosetting plastics like bakelite, insecti- decomposers is not a part.
cides like DDT, detergent, PVC, etc. These items cannot be 47. (b)
degraded biologically. 48. (c) A food chain is a linear sequence of organisms through
32. (c) The organisms in an ecosystem are related through food which nutrients and energy pass as one organism eats an-
requirements. This relationship is called food chain. In op- other.
tion (i) grass is eaten by rabbit, rabbit is consumed by wolf 49. (a)
and lion eats wolf. In option (iv) grasshopper eats grass,
frog eats grasshopper, snake eats frog and eagle consumes 50. (a)
snake. In option (ii) grasshopper is found in terrestrial eco- 51. (c) A consumer does not depend on a single organism for ful-
systems not aquatic ecosystems. In option (iii) there is no filling its energy requirements. It may eat the organism from
herbivore animal to consume grass. a different food chain, thus interlinking them to form food
33. (a) 1% energy that is absorbed by green plants for photosyn- webs.
thesis as even 1% of the energy from the sunlight falling on 52. (a)
plant leaves is enough to excite the core of photosystems
53. (b)
and the electrons move to the electron transport system be-
ginning the photosynthesis process. 54. (c)
 There are countless leaves on green plants and an entire 55. (b) Plants are producers, ants eat plants, birds eat both plants
day of sunlight is what plants receive normally any more and ants, snakes will eat the birds and the lion preys on
energy would actually destroy the photosystems core and them.
cause damage to the leaves. 56. (b) The greenhouse effect is a natural process that warms the
34. (d) Biological magnification is the process in which the chem- Earth’s surface. When the Sun’s energy reaches the Earth’s
icals supplied as fertilizers or pesticides during agricultural atmosphere, some of it is reflected back to space and the
processes enter into the plants along with water through rest is absorbed and re-radiated by greenhouse gases. The
their roots. These are retained by the fruits and seeds which absorbed energy warms the atmosphere and the surface of
are consumed by humans or animals. the Earth.
BIOLOGY 49

57. (a) A – (p); B – (q); C – (r); D – (s) 76. (a) Crown fires are most destructive. These destroy the cano-
58. (b) A – (r); B – (s); C – (p); D – (q) pies of trees and entire vegetational cover of that particular
area. Due to this fire, the temperature, rises upto 700˚C and
59. (c) A – (s); B – (r);C – (q); D – (p)
may adversely effect the nearly vegetation.
60. (d) A – (q); B – (s); C – (p); D – (r)
77. (a) The assertion is correct because plants can synthesize food
61. (a) A – (r); B – (p); C – (s); D – (q) due to the presence of abiotic components like carbon di-
62. (a) Biodegradable wastes should be separated and kept in oxide and sunlight, due to which energy is produced. This
green colour bins for garbage collectors. energy is transferred at each trophic level. The reason is
correct because due to energy obtained, organisms at each
63. (a) CFCs (chloro-fluoro-carbons) causes depletion of ozone
trophic level sustain.
layer thereby allowing more UV-radiations to reach the
earth. 78. (a)

64. (c) 79. (d) Each step or division in food chain which is characterised
by a particular method of obtaining food is called trophic
65. (c)
level. A food chain usually possesses 3-4 trophic levels like
66. (c) producers e.g., plants, herbivores e.g., rabbit, grasshopper,
67. (d) The abiotic components of the environment are the nonliv- carnivores e.g., frog, fox and secondary carnivores e.g.,
ing factors like temperature, pressure, rainfall, soil, valley, Hawk etc.
mountain and plain etc. Ecosystem consists of a distinct bi- 80. (a) All the interacting organisms in an area together with the
otic community and exchanging material between them. non-living constituents of the environment form an eco-
68. (d) Energy flow is the key function in an ecosystem and it is system. Thus, an ecosystem consists of biotic components
unidirectional. Ten percent law states that during transfer of comprising living organisms and abiotic components com-
food energy, only 10% is stored at higher trophic level. The prising physical factors like temperature, rainfall, wind, soil
remaining 90% is lost during feeding, digestion and respira- and minerals.
tion. 81. (a) Ozone depletion, gradual thining of earth’s ozone layers in
69. (c) the upper atmosphere caused by the release of chemical
compounds containing gaseous chlorine or bromine from
70. (d) Ten percent law (Lindeman, 1942) states that during trans-
industry and other human actualities. The thining is most
fer of food energy from one trophic level to the next one,
pronounced in the Polar regions, especially over Antarctica.
only ten percent is stored at higher trophic level. The re-
maining 90% is lost during feeding, digestion and respira- 82. (c) Tropical rain forests have disappeared mainly due to man’s
tion. activities. Due to over population in countries like India,
rain forests are cut to make place available for man to live
71. (a) Functioning of an ecosystem requires a flow of energy. Its and build houses. To build buildings and factories man has
movement is unidirectional from sun to plants, plants to an- incessantly cut down trees. This has caused the depletion of
imals, animals to animals, organic remains to decomposers rain forests.
and dissipation as heat. 83. (a) Animals blend with the surroundings or background to re-
72. (a) The living components of an ecosystem are arranged in a main unnoticed for protection and aggression.
hierarchical order with respect to food dependency. In a 84. (a) Pyramid of biomass is a graphic representation of total
amount of organic matter in an ecosystem at any time per
food chain, the organisms present in first trophic level are
unit area at different trophic levels.
producers which synthesise food for themselves as well as
85. (b) Ecosystem is a self-sustaining structural and functional unit
all the other organisms.
of the biosphere.
 At the second trophic level, are present herbivores which 86. (a) Food web is a network of food chains operating in an eco-
consume producers and at the third trophic level are pres- system which get connected at various trophic levels.
ent carnivores which consume herbivores for their nutrition 87. (a) Bioremediation is a technology for cleaning the environ-
supply. ment with biological options such as microbes and plants.
73. (c) Biodegradable wastes are wastes which are broken down 88. (b)
and disposed off naturally by saprophytes or decomposers 89. (d)
74. (b) Non-biodegradable are wastes which cannot be broken 90. (c)
down by decomposers because they do not have enzymes 91. (c) Global warming have the following properties
for the same. i. The concentration of greenhouse gases grows, more
75. (c) Non-biodegradable substances like pesticide residues can- heat is trapped in the atmosphere and less escapes
not be digested or consumed by organisms, so, they are back into space.
just passed on to organisms of next trophic level along with ii. It changes the climate and alters weather patterns,
food. These non-biodegradable residues keep on accumu- which may hasten species extinction, influence the
lating. Higher the trophic level an organism occupies in the length of seasons, cause coastal flooding etc.
food chain higher will be the accumulation of non-biode-
gradable pesticide residues.
Notes

You might also like